Sei sulla pagina 1di 244
434 Chapter 6 The Frequeney-Response Design Method EXAMPLE 6.18 Note that Examples 6.15 and 6.17 are both for the same plant and both had the same steady-state error requirement. One was compensated with lead and one was compensated with lag. The result is that the bandwidth of the lead compensated design is approximately a factor of 3 higher than the lag compen- sated design, ‘This result can be seen by comparing the crossover frequencies Of the two designs ‘A beneficial effect of lag compensation, an increase in the low-frequency gain for better error characteristics, was just demonstrated in Example 6.17. However, in essence, lag compensation reduces the value of ;2, (= 3 fora type 1 system). That means that, if the crossover frequency is kept the same, the low frequency gain will increase. Likewise, ifthe low frequency gain is kept the same, the crossover frequency will decrease, Therefore, lag compensation could also be interpreted to reduce the crossover frequency and thus obtain a better phase margin. The procedure for design in this case is partially modified, First, pick the low-frequency gain to meet error requirements, then locate the lag compensation pole and zero in order to provide a crossover frequency with adequate PM. The example below illustrates this design procedure, The end result of the design will be the same no matter what procedure is followed. Lag Compensation of the DC Motor Repeal the design of the DC motor control in Example 6.14, this time using lag eompen- sation. Fix the low-frequency gain in order to meet the error requirement of K, = 10; then use the lig compensation to meet the PM requirement of 45 Solution, The frequency response of the system KG(s), with the required gain of K = 10, isshown in Fig. 665. The uncompensated system has a crossover frequency at approximately 3 radisee where the PM = 20°. The designer's task is 1 select the lag compensation break points so thatthe crossover frequency is loweged and more favor able PM results. To prevent detrimental effects from the compensation phase lap, the ppole and zero position values of the compensation need to be substantially lower than the new crossover frequency, One possible choice is shossn in Fig. 665; The lag zero is At .L radisec, andthe lag pole is at 0.01 radisce. This selection of parameters produces PM of 50°, thus satisfying the specifications Here the stabilization is achieved by keeping the crossover frequency to a region where Gis) has favorable phase character istics. The criterion for selecting the pole and zero locations 1/7 is to make them low enough to minimize the effects ofthe phase lag from the compensation atthe crossover frequency. Generally, however, the pole and zero are located no lower than necessary because the additional system root [compare with the root locus of a similar system design in Fig, 5.31(b)] introduced by the lag will be inthe same frequency range as the compensation zero and will have some effect on the output response, especially the response to disturbance inputs. The response ofthe system to a step reference input is shown in Fig. 666. It shows, no steady-state error to a step input because this is a type | system. However, the introduction of the slow root from the lag compensation has caused the response 10 require about 25 sec to settle down to the zero steady-state value. The overshoot, My, is somewhat larger than you would expect {rom the guidelines based on a second-order system shown in Fig 6.37 for a PM = 50°: however, the performance is adequate, Figure 6.65 Frequency response of lag-compensation design in Example 6.18 Figure 6.65 Step response of lag-compensation design in Example 6.18 Section 6.7 Compensation 435 === KDWNGE) — Ka) ae q 2 0 ne Goor oor v1 02 tradhes) \, 90" oe © iso ath PM — 50 pM = 20 -a | ‘oor wor oro2 1 2 eteadsec os 04 S018 29 3 Ww aS Was 50 As we saw previously for a similiar situation, Examples 6.14 and 6.18 meet an identical set of specifications for the same plant in very different ways. In the first case the specifications are met with a lead compensation, and a crossover frequency w, = 5 radisec (wpy = 6 radisec) results. In the second case the same specifications are met with a lag compensation, and «, = 0.8 rad/sec 436° Chapter 6 in eamnensation EXAMPLE 6.19 The Frequency-Response Design Method (waw * 1 radisec) results. Clearly, had there been specifications for rise time or bandwidth, they would have influenced the choice of compensation (lead or lag). Likewise, if the slow settling to the steady-state value was a problem, it ‘ight have suggested the use of lead compensation instead of lag. In more realistic systems, dynamic elements usually represent the actuator and sensor as well as the process itself, so it is typically impossible to raise the crossover frequency much beyond the value representing the speed of response of the components being used. Although linear analysis seems to suggest that almost any system can be compensated, in fact, if we attempt to drive a set of components much faster than their natural frequencies, the system will saturate, the linearity assumptions will be no longer valid, and the linear design will represent little more than wishful thinking, With this behavior in mind, we see that simply increasing the gain of a system and adding Iead compensators to achieve an adequate PM may not always be possible. It may be preferable to satisfy error requirements by adding a lag network so that the closed-loop bandwidth is kept at a more reasonable frequency. 6.7.5 PID Compensation For problems that need phase margin improvement at w, ant low-frequency gain improvement, its effective to use both derivative and integral control. By combining Eqs. (6.37) and (6.50), we obtain PID control. Its transfer function i=" fro en(o+ 2). os and its frequency-response characteristics are shown in Fig. 6.67. This form is slightly different from that given by Eq. (4.44): however, the effect of the differ- ence is inconsequential. This compensation is roughly equivalent to combining lead and lag compensators in the same design, and so itis sometimes referred to asallead-lag compensator. Hence, it can provide simultaneous improvement in transient and steady-state responses, PID Compensation Design for Spacecraft Atitude Control A simplified design for spaceeraftattitude-control was presented in Seotion 6.5: how- fever, here we have a more realistic situation that ineludes a sensor lag and a disturbing torque. Figure 6.68defines the system. Design a PID controller have 2erosteady-state error to a constant-disturbance torque, a phase margin of 65°. and as high a bandwith as is rexsonably possible Solution, First, let us take care of the steady-state error. or the spacecraft to be ata steady final Value, the (otal input torque, Ty + T., must equal zero, Therefore, if Ty) #0. then T, = —Ty. The only way this can be true with no error (e = 0) is for Dis} to contain an integral term, hence including integral control in the compensation will meet the steady-state requirement. This could also be verified mathematically by use of the Final Value Theorem (see Problem 6.46), Figure 6.67 Frequency response ot ID compensation with Ti/Tp = 20 Figure 6.68 Bock diagram of Spacecraft Contra using PID design, Example 6.19 Section 6:7 Compensation 437 10K Deal 2x oroz oo or ww 20) ar ~0 wwe bert | | it oor 4 0 100 “ ‘The frequency response of the spacecraft and sensor, ow ="2(25) os is shown in Fig, 6.69. The slopes of ~2 (that is, —40 db per decade) and —3 (60 db peer decade) show that the system would be unstable for any value of K if no derivative Feedback were used, This is clear because of Bode'sgain-phase relationship which shows n—$(E)—}ow LS 20 438 Chapter 6 The Frequency-Response Design Method Figure 6.68 Compensation for PID esign in Example 6.19 0 a Slope iE a von oor art 2 No Noo aceadee) ‘ DinGe, fe = 00 Dosyain. Fe 04 2a op Leet door aor ar tries thatthe phase would be ~180° for the -2 slope and —270° for the ~3 slope and whieh would correspond to a PM of 0° or 90". Therefore, derivative control is required (0 bring the slope to ~1 at the crossover frequency that was shown in Section 6.5 to be a requirement for stability. The problem now is to pick values for the three parameters in Eq. (6.52)—K., Tr, and T;—that will satisfy the specifications. The easiest approach isto work frst on the phase so that PM = 65° isachieved ata reasonably high frequeney. This ean be accomplished primarily by adjusting Ti, noting that 7; has a minor effect if suficiently larger than Ti. Once the phase is adjusted, we cstablish the crossover lrequeney and then can easily determine the gain K Section 6.7 Compensation 439 We examine the phase of the PID controller in Fig. 6.67, to determine what would happen to the compensated spacecraft system. D(s)G(s), a8 To is varied. If 1/Tp > 2 radisee, the phase lead from the PID control would simply cancel the sensor phase lag, and the composite phase would never exceed —180, an unacceptable situation. If 1/Tp = 0.01, the composite phase would approach 90° Tor some range of frequencies and would exceed ~115° for an even wider range of frequencies; the latter threshold \would provide a PM of 65". In the compensated phase curve shown in Fig. 669, 1/ Tp, 0.1, whieh isthe largest value of 1/7, that could provide the required PM of 65°, The phase would never ross the —115° (65°PM) line for any L/Tp = @.1. For 1/Tp = 0.1 the crossover frequeney «that produces the 65°PM is 05 radisee. For a value of 1/Ty < 0.05, the phase essentially follows the dotted curve in Fig. 669, which indicates that the maximum possible «is approximately | radisee and is provided by 1/To = 0.05. Therefore, 0.05 < 1/To < 0.1 is the only sensible range for 1/To; anything less than 0.05 would provide no significant inerease in bandwidth, while anything more than 0.1 could not meet the PM specification. Although the final choice is somewhat arbitrary, we have chosen 1/Ty =0.1 for our final design. Our choice for 1/7; is a factor of 20 lower than I/To: that is, 1/7) = 0.005. A factor less than 20 would negatively impact the phase at crossover, thus lowering, the PM. Furthermore, itis generally desirable to keep the compensated magnitude as large as possible at frequencies below w, in order to have a faster transient response and smaller errors: maintaining 1/7), and 1/7; at the highest possible frequencies will bring this about, “The only remaining task is to determine the proportional part of the PID controller, or K. Unlike the system in Example 6.1, where we elected K inorder tomeetasteady. state error specification, here we elect a value of K that will yield a crossover frequency at the point corresponding tothe required PM of 65°. The basie procedute for finding K discussed in Section 646, consists of plotting the compensated system amplitude with K’= 1, finding the amplitude value at crossover, then setting 1/A equal to that value, Figure 6.69 shows that when = 1, |D(s)G(s)) = 20 atthe desired crossover frequency fu = 05 tadisee, Therefore, =, 30 ‘The compensation equation that stisies al ofthe specifications is now complete 008 15) = “Bjads + 1)¢¢+ 0.005) Icisinteresting to note that this system would become unstable if the gain were low: tered so that a, = 0.02 radisec the region in Fg. 6.69 where the phase ofthe compensated system is less than —180°. As mentioned in Section 6.4, this situation is referred to asa conditionally stable system. A root locus with respect o K’for this and any condition: ally stable system would show the portion of the locus corresponding to very low gains in the RHP. The response of the system fora unit step Sym is shown in Fig 6.70(a) and exhibits well-damped behavior, as should be expected with a 65” PM, The response of the system for a step disturbance torque T = 0.1 N is shown in Fig. 6.70(b). Note thatthe integral control term does eventually drive the error to zer0: however, it isslow du tothe presence of a closed-loop pole inthe vicinity of the zero at (0.005, However, recall [rom the design process that this zero was located in order ‘that the integral term not impact the PM unduly. So ithe slow disturbance response is not acceptable, speeding up this pole will decrease the PM and damping of the system. ‘Compromise is often a necessity in control system design! 440. Chapter 6 ‘The Frequency-Response Design Method 12 oe 1.80 10 + 7 os 120 , 100 | #06 ie os 0% ae ao ee 020 © 0 0 ws sO (0100 00 308 300 S09 G1? 700-8) 900 TOR Time se) ” Figure 6.70 Transient response for PID example: (a) step response; (b)step-dsturbance response Summary of Corppnsation (characteris We now summarize conmpensation characteristics A. PD Control sas phase lead at all frequencies above the break point. If there is no change in gain on the low-frequency asymptote, PD compensa- tion will increase the crossover Irequency and the speed of response. The increase in magnitude of the frequency response atthe higher frequencies will increase the system's sensitivity to noise 2, Lead Compensation adds phase lead ata frequency band between the two break points, which are usually selected to bracket the crossover frequency. Ifthere isno change in gain on the low-frequency asymptote, lead compen: sation will increase both the crossover frequency and the spedt of response over the uncompensated system, 3. Pl Control increases the frequency response magnitude at frequencies be ow the break point, thereby decreasing steady-state errors. It also con- tributes phase lag below the break point, which must be kept at a low ‘enough frequency to avoid degrading the stability excessively 4 Lag Compensation increases the frequency response magnitude at fre {quencies below the two break points, thereby decreasing steady-state er- ors. Alternatively, with suitable adjustments in K., lag compensation tan be used to decrease the frequeney-response magnitude at frequencies labove the two break points so that «, yields an acceptable phase margin. Lazcompensation also contributes phase lag between the two break points, ‘which must be kept at frequencies low enough to keep the phase decrease from degrading the PM excessively. This compensation will typically pro- vide a slower response than using lead compensation Figure 6.71 Etfect of high-frequency Dlant uncertainty Section 6.7 Compensation 441 6.7.6 Design Considerations We have seen in the preceding designs that characteristics of the open-loop Bode plot of the loop gain, L(s) (= K DG), determine performance with ‘respect to steady-state errors and dynamic response. Other properties of feed: back, developed in Chapter 4, include reducing the effects of sensor noise and parameter changes on the performance of the system. The consideration of steady-state errors due to command inputs and dis turbances has been an important design component in the different design methods presented. Design for acceptable steady-state errors can be thought cf as placing a lower bound on the very-low-frequency gain of the system. An- other aspect of the sensitivity issue concerns the high-frequency portion of the system, So far, Chapter 4 and Sections 5.5 and 6.7 have briefly discussed the idea that, to alleviate the effects of sensor noise, the gain of the system at high frequencies must be kept low. In fact, in the development of lead compen- sation, we added a pole to pure derivative control specifically to reduce the effects of sensor noise at the higher frequencies. It is not unusual for designers to place an extra pole in the compensation, that is, to use the relation Ts+1 De) = >= ats +1? in order to introduce even more attenuation for noise reduction. ‘A second consideration affecting high-frequency gainsis that many systems have high-frequency dynamic phenomena such as mechanical resonances that could have an impact on the stability of a system. In very-high-performance designs, these high-frequency dynamics are included in the plant model, and a compensator is designed with a specific knowledge of those dynamics. A stan: ‘dard approach to designing for unknown high-frequency dynamicsis to keep the high-frequency gain low, just as we did for sensor-noise reduction. The reason for this can be seen from the gain-frequency relationship of a typical system, shown in Fig. 6.71. The only way instability can result from high-frequency dynamics is if an unknown high-frequency resonance causes the magnitude to [+ High quency 442. Chapter 6 "The Frequency-Response Design Method Gain tabilzabon Phase stabilization Figure 6.72 Design criteria for low sensitivity rise above 1. Conversely if all unknown high-frequency phenomena are guar anteed to remain below a magnitude of 1, stability can be guaranteed. ‘The likelihood of an unknown resonance in the plant G ising above I can be re- duced ifthe nominal high-frequeney loop gain () is lowered by the addition of extra poles in Dis). When the stability of a system with resonances is as sured by tailoring the high-frequency magnitude never to exceed 1, we refer to this process as amplitude or gain stabilization. Of course, if the resonance characteristics are known exactly, a specially tailored compensation, such as fone with a notch atthe resonant frequeney, can be used to change the phase at c frequency to avoid encirclements of —1, thus stabilizing the system. even though the amplitude does exceed magnitude 1. This method of stabilza- tion is referred to as phase stabilization. A drawback to phase stabilization is that the resonance information is often not available with adequate precision ‘or varies with time: therefore, the method is more susceptible to errors in the plant model used in the design, ‘Thus we see that sensitivity to plant uncer- tainty and sensor noise are both reduced by sufficiently low loop gain at high frequency These two aspects of sensitvity—high- and low-frequency behavior—can be depicted graphically asshown in ig. 6.72, There sa minimum low-frequency gain allowable for acceptable steady:state error performance and a maximum high-frequency gain allowable for acceptable noise performance and for low probability of instabilities caused by plant-modeling errors. Itissometimescon ‘enient to define the low-frequency lower bound onthe frequency response as W; and the upper bound as W,! as shown in the figure. Between these two bounds the control engineer must achieve a gain crossover near the required bandwidth; as we have seen, the crossover must occur at essentially a slope of =I for good PM and hence damping For example, if control system was required to follow a sine reference in put with frequencies from 0 to wy with errors no greater than 1%, the function WW, would be 100 from «= 0 to a. Similiar ideas enter into defining possible values for the W! function. These ideas will be discussed further in Section 6.9. ‘Sensor noite and plant uncertainty boundary SteadyState ror boundary Mand Wires Section 6.8 Alternate Presentations of Data 443 & 6.8 Alternate Presentations of Data Before computers were widely available, other ways to present frequency response data were developed to aid both in understanding design and in easing the designer's work load. The widespread availability of computershas reduced the need for these methods. Two techniques are the Nichols chart and the in- verse Nyquist plot, both of which we examine in this section because of their place in history. 6.8.1 Nichols Chart A rectangular plot of log |G(jo)] versus £G(ja) can be drawn by simply trans- {erring the information directly from the separate magnitude and phase por tions in a Bode plot; one point on the new curve thus results from a given value of the frequeney «o. This means that the new curve is parameterized as a func- tion of frequency. As with the Bode plots, the magnitude information is plotted on a logarithmic scale, while the phase information is plotted on a linear scale. This template was suggested by N. Nichols and is usually referred to as.a Nichols chart. The idea of plotting the magnitude of G(jo) versus its phase is similar to the concept of plotting the real and imaginary parts of G(jw), which formed the basis for the Nyquist plots shown in Sections 6.3 and 64. However, it is difficult to capture all the pertinent characteristics of G( jw) on the linear scale of the Nyquist plot. The log scale for magnitude in the Nichols chart alleviates, this difficulty, allowing this kind of presentation to be useful for design, For any value of the complex transfer function G( ja), Section 6.6 showed that there isa unique mapping to the unity-feedback closed-loop transfer func- on Ge) - iw) 054 ue) 1+G(jo) (654) or, in polar form, T(ja) = Mwyel™ (655) Where M(o) is the magnitude of the closed-loop transfer function and e(o) is the phase of the closed-loop transfer function. Specifically, (6.56) @ = tan WN) (657) +6 It can be proven that the contours of constant closed-loop magnitude and phase are circles when G(jo) is presented in the linear Nyquist plot. These circles are referred to as the M and N circles, respectively. 444 Chapter 6 The Frequency-Response Design Method s Lon 4 5 | — > Closed. loop rmagniude eI or 20K9G0) Figure 6.73 Nichols chart The Nichols chart also contains contours of constant closed-loop magnitude and phase based on these relationships, as shown in Fig, 6.73; however, they are no longer circles because the Nichols charts are semilog plots of magnitude versus phase. A designer can therefore graphically determine the bandwidth of a closed-loop system from the plot of the open-loop data on a Nichols chart by noting where the open-loop curve crosses the 0.70 contour of the closed-loop magnitude and determining the frequency of the corresponding data point. Likewise, a designer can determine the resonant peak amplitude M, by noting the value of the magnitude of the highest closed-loop contour tangent to the curve. The frequency associated with the magnitude and phase at the point of Resonant frequency tangency is sometimes referred to as the resonant frequency «,.. Similarly, a designer can determine the gain margin (GM) by observing the value of the gain Section 6.8 Alternate Presentations of Data 445 where the Nichols plot crosses the ~180° line, and he or she can determine the phase margin (PM) by observing the phase where the plot crosses the amplitude I ine (James etal., 1947). Marian provides for easy drawing of a Nichols chart the nichols function, EXAMPLE 6.20. Nichols Chart for PID Example Determine the bandwith and resonant peak magnitude of the compensated system whose frequeney response is shown in Fig. 6.9. Solution, The magnitude and phase information of the compensated design example seen in Fig. 6.69 is shown on a Nichols chart in Fig. 674. When comparing the two figures, iti important to divide the magnitudes in Fig. 6.69 by a factor of 20 in order FS chose Joop magnitude Closet loop hs 160" 130° nao" 00" ‘0° 7 0° DEG Figure 6.74 Example plot on the Nichols chart for determining bandwidth and, 446 Chapter6 The Frequency-Response Design Method EXAMPLE 6.21 to obtain |D(s)G(s)| rather than the normalized values used in Fig, 6.69. Because the ‘curve crosses the closed-loop magnitude 0.70 contour at w = 0.8 radisee, we see that the bandwidth of thissystem 0.8 rales, Bocauxe the largest-magnitude contour touched by the curve is 1.20, we also see that M, This presentation of data was particularly valuable when a designer had to generate plotsand perform calculations by hand. A change gain, for example, could be evaluated by sliding the curve vertically on transparent paper over a standard Nichols chart. The GM, PM, and bandwidth were then easy to read off the chart, thus allowing evaluations of several values of gain with a minimal amount of effort. With access to computer-aided methods, however, we can now calculate the bandwidth and perform many repetitive evaluations of the gain or any other parameter with a few key strokes, Today the Nichols chart is Used primarily as an alternate way to present the information in a Nyquist plot, For complex systems where the ~1 encirclements need to be evaluated, the ‘magnitude log scale of the Nichols chart enables us to examine a wider range of frequencies than a Nyquist plot does, as well as allowing us to read the gain and phase margins directly. Stability Margins from Nichols Chart For the system of Example 6.12, whose Nyquist plot is shown in Fig. 6.41, determine the PM and GM using the Nyquist plot Solution, — Figure 6.75 shows a Nichols chart with the daa from the same system shown, in Fig. 641. Note that the PM for the magnitude I crossover frequeney is 36° and the GMis 1.25 (= 1/08). It isclear from this presentation ofthe data that the most critical portion ofthe curve is where it crosses the — 180” line; hence the GMs the most relevant Stability margin in this example, és 6.8.2. Inverse Nyquist ‘The inverse Nyquist plot is simply the reciprocal of the Nyquist plot described in Section 6.3 and used in Section 6.4 for the definition and discussion of stability margins. It is obtained most easily by computing the inverse of the magnitude from the Bode plot and plotting that quantity at an angle in the complex plane as indicated by the phase from the Bode plot. It can be used to find the PM and GM in the same way that the Nyquist plot was used. When |G] 1, |G-"(je)| = 1 also, so the definition of PM is identical on the two plots. However, when the phase is ~180° or +180", the value of |G~"(jo)| is the GM directly; no calculation of an inverse is required, as was the case for the Nyquist plot The inverse Nyquist plot for the system in Fig. 6.24 (Example 6.9) is shown in Fig, 6.76 for the case where K = 1 and the system is stable, Note that 10 ox 07 ta Figure 6.75 Nichols chart of the complex system in Examples 6.12 and 6.21 Figure 6.76 Inverse Nyquist plot for Sample 6:9 ae 20 iar T08 2061618) fimo Ls to os 05 to 1s or tion 68 Alternate Presentations of Data 447 ~ Closed magnitude 448 Chapter 6 The Frequency-Response Design Method Figure 6.7 Inverse Nyquist plat ofthe system whose Nyquist plot is in Fig, 641 Ino“h) ele" GM = 2 and PM = 20°, Asan example of a more complex case, Fig. 6.77 shows an inverse Nyquist plot for the sixth-order case whose Nyquist plot was shown in Fig. 6.41 and whose Nichols chart was shown in Fig. 6.75, Note here that GM = 1.2 and PM Had the two crossings of the unit circle not occurred at the same point, the crossing with the smallest PM would have been the appropriate one 10 use. 6.9 Specifications in Terms of the Sensitivity Function We have seen how the gain and phase margins give useful information about the relative stability of nominal systems and can be used to guide the design of lead and lag compensations. However, the GM and PM are only two numbers and have limitations as guides to the design of realistic control problems. We ‘ean express more complete design specifications in the frequehey domain if we frst give frequeney descriptions for the external signals such as the reference and disturbance and consider the sensitivity function defined in Section 43, For example, we have so far described dynamic performance by the transient response to simple steps and ramps. A more realistic description of the actual complex input signals is to represent them as random processes with corre- sponding frequency power density spectra. A less sophisticated description which is adequate for our purposes isto assume that the signals can be repre- ented as a sum of sinusoids with frequencies ina specified range. For example, wwe can usually deserie the frequency content ofthe reference input as a sum ‘of sinusoids with relative amplitudes given by a magnitude function [| such as that plotted in Fig. 6.78 which represents a signal with sinusoidal components having about the same amplitudes up to some value « and very small ampli- tudes for frequencies above that. With this assumption the response tracking specification can be expressed by’a statement such as “the magnitude of the system error is to be less than the bound ¢» (a value such as 001) for any si nusoid of frequency «yin the range 0 w, = a and of amplitude given by Figure 6.78 Plot of typical reterence spectrum Sonstiay tution Figure 6.79 Ciosed-ioop block diagram Section 6.9 Specifications in Terms of the Sensitivity Function 449 © lal w ccadece) R( jor). To express such a performance requirement in terms that can be used in design, we consider ayaitt te unity feedback syste drawn int Fig. 6.79. For this system the error is given by (6.8) (6.59) In addition to being the factor multiplying the system error, the sensitivity function is also the reciprocal of the distance of the Nyquist curve, DG, from the critical point —1. A large value for S indicates a Nyquist plot that comes close to the point of instability. The frequency-based error specification based on Eq. (6.58) can be expressed as |£| = |S||R] < e». In order to normalize ow 450 Chapter 6 The Frequeney-Response Design Method the problem and not need to define both the spectrum R and the error bound each time, we define the real function of frequency Wy(w) = |R| /ep and the requirement can be written as Wt] (6.60) EXAMPLE 6.22 Performance Bound Function ‘A unity feedback system is to have an error less than 0.005 for all unity amplitude sinusoids below frequency 100 Hz, Draw the performance frequency function W;(o) for this design, Solution, The spectrum, from the problem description, is unity for 0 < « = 200%. Becattse ey = 0.005, the required functions given by a rectangle of amplitude 10.005 = 200 over the given Fange. The function is plotted in Fig. 6.80. Figure 6.80 tot 7° Plot of example performance function, Wi ww 10! 2 w ‘ w a Figure 6.81 4) a Prot of typical pant uncertainty, W2 q 0 0" 0 ri v0! sateadsce) ‘The expression in Eq, (6,60) can be translated to the more familiar Bode plot coordinates and given as a requirement on loop gain by observing that over the frequency range when errors are small, the loop gain is large. In that case |$| ~ 1/ |DG| and the requirement is approximately ™ sl (661) Section 6.9 Specifications in Terms of the Sensitivity Function 451 ‘This requirement can be seen as an extension of the steady-state error require- ‘ment from just @ = 0 to the range 0 < @, < a In addition to the requirement on dynamic performance, the designer is usually required to design for stability robustness, By this we mean that, while the design is done for a nominal plant transfer function, the actual system is expected to be stable for an entire class of transfer functions that represents the range of changes that are expected to be faced as temperature, age, and other operational and environmental factors vary the plant dynamics from the nominal case. A realistic way to express this uncertainty is to describe the plant transfer function as having a multiplicative uncertainty as GJ) = GoJo + WelordGoo)} (6.62) In Eq. (6.62), the real function W2 is a magnitude function that expresses the size of changes as a function of frequency that the transfer function is expected to experience. In terms of G and G, the expression is |G-G, (6.03) The shape of W is almost always very small for low frequencies (we know the model very well there) and increases substantially as we go to high fre quencies where parasitic parameters come into play and unmodeled structural flexibility is common. A typical shape is sketched in Fig. 6.81. The complex function, A(je), represents the uncertainty in phase and is restricted only by the constraint 0<|Als1 (6.654) 1 © Ww | 40 | | wo! | 20 wo - ° wt 20 wo ~ 40 0 1 ro 10! 10" trade 452. Chapter 6 EXAMPLE 6.23 The Frequency-Response Design Method We assume that the nominal design has been done and is stable so that the Nyquist plot of DG, satisfies the Nyquist stability criterion. In this ease, the nominal characteristic equation 1 + DG, = 0 is never satisfied for any real fre- quency. If the system is to have stability robustness, the characteristic equation using the uncertain plant as described by (6.62) must not go to zero for any real frequency for any value of A. The requirement can be written as 14+ DG #0. 1406, 1+ Waa] 40 (6.065) (1+ DG) TWA) 40, where we have defined the complementary se ity function as T(s) 2 DG,/( + DG,) 5. (6.66) Because the nominal system is stable, the first term in Eq, (6.65), (1 + DGy), is never zero, Thus, if Eq. (6.65) is not to be zero for any frequency and any A, then it is necessary and sufficient that IrWeal <1 which reduces to TIM <1, (6.67) making use of Eq. (6.64). As with the performance specification, for single- input-single-output unity feedback systems this requirement can be approxi mated by a more convenient form, Over the range of high frequencies where W5 is non-negligible because there is significant model unce'tainty. DG, is small. Therefore we can approximate T ~ DG, and the constraint reduces to [DGo| Wa — (6.68) Typical Plant Uncertainty The uncertainty in a plant model is described by a function W; which is zero until = 3000 and increases linearly from there toa value of 100 at «» = 10,000 and remains fat 100 for higher frequencies, Plot the constraint on DG, to meet this requirement. Solution, Where W: = 0, there is no constraint on the magnitude of loop gain: above = 3000, 1/1; = DG, isi hyperbola from co to 0.01 at = 10,0000 and remains at (0.01 for > 10,000, The bound is sketched in Fig. 6.32. Figure 6.82 Prot of constraint on [DGol (= |W>'D Figure 683 Tracking and stabity robustness constraints on the Bode plot Section 6,9 Specifications in Terms of the Sensitivity Function 453 w 40 I 10! + 20 10" 20 woe 40 10 107 107 vw wtadsee) In practice, the magnitude of the loop gain is plotted on log-log (Bode) coordinates and the constraints of Eq, (6.61) and Eq, (6.68) ate included on the same plot. A typical sketch is drawn in Fig, 6.83. The designer is expected to 0 10! 420 w g wt -m a a a we ue Swe vot eteadec) 454 Chapter 6 The Frequency-Response Design Method EXAMPLE 6.24 construct a loop gain that will stay above W; for frequencies below «1, cross over the magnitude 1 line (DG| = 0) in the range @ < @ < wa, and stay below 1/W) for frequencies above w, 6.9.1 Limitations on Design in Terms of the Sensitivity Function One of the major contributions of Bode was to derive important limitations on transfer functions that set limits on achievable design specifications. For example, one would like to have the system error kept small for the widest possible range of frequencies and yet to have a system that is robustly stable fora very uncertain plant. In terms of the plot in Fig. 6.83, we want W; and W to be very large in their respective frequency ranges and for 1 to be pushed up close to w». Thus the loop gain is expected to plunge with a large negative slope from being greater than W; to being less than 1/W in a very short span, while maintaining a good phase margin to assure stability and good dynamic performance. The Bode gain-phase formula given earlier shows that this is impossible with a linear controller by showing that the minimum possible phase is determined by an integral depending on the slope of the magnitude curve, Ifthe slope is constant for a substantial range around ,, then the formula can be approximated by x dM (on) => (6.69) Ho) = 5 ) where M is the log magnitude and w = log. If, for example, the phase is to be kept above —150° to maintain a 30° phase margin, then the magnitude slope near «, is estimated to be Tem) = 21667 If we try to make the average slope faster (more negative) than this, we will lose the phase margin. From this condition there developed the design rule that the asymptotes of the Bode plot magnitude, which are restricted to be integral values for rational functions, should be made to cross over the O-db line at @ slope of ~1 over a frequency range of about one decade around the crossover frequency as already discussed in Section 6.5. Modifications to this rule need of course be made in particular cases, but the limitation implied by Eq. (6.69) isa hard limit that cannot be avoided. Robustness Constraints Ww, 30°, what i the minimum ratio of «n/n? = 100, and we want PM. Section 6.9 Specifications in Terms of the Sensitivity Function 455 Solution, The slope is toto 242 Th Thus the log of the ratio is log {8 = —2.40 and «> = 25ter An alternative to the standard Bode plot as a design guide can be based on a plot of the sensitivity function as a funetion of frequency. In this format, Eq. (6.60) requires that |S) < 1/Wy over the range 0 < w = w for perfor ‘mance and Eq, (6.68) requires that |$| = I over the range a < for stability robustness. It should come as no surprise that Bode found a limitation on the possibilities in this case, too, The constraint, extended by Freudenberg and Looze (1985), shows that an integral of the sensitivity function is determined. by the presence of poles in the right half-plane. Suppose the loop gain DG,, has mp poles, pin the right half-plane and “rolls off” at high frequencies at a slope faster than ~1. For rational functions, this means that there is an excess of at least two more finite poles than zeros, ‘Then it can be shown that Jf inispde Ifthere are no right half-plane poles, then the integral iszero. ‘Thismeans that if ‘we make the log of the sensitivity function very negative over some frequency band to reduce errors in that band, then of necessity In|S| will be positive over another part of the band and errors will be amplified there. If there are unstable poles, the situation is worse because the positive area where sensitivity ‘magnifics the error must exceed the negative area where the error is reduced by the feedback. If the system is minimum phase, then itis in principle possible to keep the magnitude of the sensitivity small by spreading the sensitivity increase over all positive frequencies to infinity but such a design requires an excessive bandwidth and is rarely practical. If a specific bandwidth is imposed, then the sensitivity function is constrained to take on a finite, possibly large, positive value at some point below the bandwidth. As implied by the definition of the vector margin (VM) in Section 6.4, (Fig, 6.38), a large Syyax corresponds to Nyquist plot that comes close to the ~1 critical point and a system having a small vector margin since 7S Rein} (670) Sax vM (671) Sax ~ Ifthe system is not minimum phase, the situation is worse. An alternative to Eq. (6.70) is true if there is a nonminimum-phase zero of DG, a zero in the right half-plane. Suppose the zero is located at zu = 0. + jw,, where 6» > 0. Again we assume there are my right half-plane poles at locations p, 456 Chapter 6 The Frequeney-Response Design Method EXAMPLE 6.25 Figure 6.84 Sensitivity function for Example 625 with conjugate values 77. Now the condition can be expressed as a two-sided weighted integral e o wo) PEt Ins) dw = Yk 672) [Loraremare acl 4 In this case, we do not have the “roll-off” restriction and there is no possibility of spreading the positive area over high frequencies because the weighting Function goes to zero with frequency. The important point about this integral is that if the nonminimum-phase zero is close (o a right half-plane pole, the right side of the integral can be very large and the excess of positive area is required to be correspondingly large. Based on this result, one expects especially great difficulty meeting both tracking and robustness specifications on sensitivity with, a system having right half-plane poles and zeros close together. Sensitivity Function for Antenna ‘Compute and plot the sensitivity function for the design ofthe antenna for which G(s) = Ts(s +1) and D(s) = 1005s + 1)/(.1s +1) Solution. The sensitivity function for this ease is 2 eet or s+ Us? + 60s + 100 and the plot shown in Fig. 684 is given by the MATLA commands 1111 100; dens = [1 11 60 100); sys$ = tinumS, dens}; ‘ [mag,ph.a] = bodelsyss} semilogy(w.mnag) ums: > so 2s magnituse Time-delay phase Figure 6.85 Phase fag duo to pure tine delay Section 6.10 Time Delay 457 The largest value of Sis given by At the vector margin is VM = 3.73, max(mag) and is given by 1.366 from which 6.10 Time Delay The Laplace transform of a pure time delay is G(s) °Ty and was ap- proximated by a rational function (Padé approximate) in our earlier diseussion of root-locus analysis Although this same approximation could be used with frequeney-response methods, an exact analysis of the delay is possible withthe Nyquist riterion and Bode plots The frequency response ofthe delay is given by the magnitude and phase of ajo. The magnitude is (Go(ja)| = [e-!™| = 1 forall. (6.74) This result is expected, because a time delay merely shifts the signal in time and has no effect on its magnitude. The phase is, LG pljo) = ~0Ty (675) in radians, and it grows increasingly negative in proportion to the frequency. This, too, is expected since a fixed time delay 7, becomes a larger fraction or multiple of a sine wave as the period drops due to increasing frequency. A plot of 2G p(ja) is drawn in Fig. 6.85. Note that the phase lag is greater than 270° for values of wf, greater than about 5 rad. This trend implies that it would be virtually impossible to stabilize a system (or to achieve a positive PM) with a crossover frequency greater than @ = 5/T,, and it would be difficult for frequencies greater than w = 3/Ty. These characteristics essentially place a constraint on the achievable bandwidth of any system with a time delay. (See Problem 6.68 for an illustration of this constraint.) ‘The frequency-domain conceptssuch as the Nyquist criterion apply directly to systems with pure time delay. This means that no approximations (Padé type or otherwise) are needed, as shown in the following example ° 0 =270 Golo 360" 4507 sur ox — or oF 110100 19H or ea 458 Chapter 6 The Frequency-Response Design Method EXAMPLE 6.26 Figure 6.86 Nyquist plot for Example 6:26, Nyquist Plot for System with Time Delay Consider the system with si 2 KE KG) = where Ty = I see, Determine the range of K for which the system is stable. Solution, Because the Bode plotting rules do not apply for the phase of a time delay term, we will se an analytical approach to determine the key features of the Irequeney response plot. As just discussed, the magnitude of the frequency response of the delay terms unity and its phase is ~« radians. The magnitude ofthe frequency response of the pure integrator is 1 /« with a constant phase of ~7/2. Therefore, Giw) = Lesin ~ jose (0.76) Using Bq, (676) and substituting in different values of o, we can make the Nyquist plot, which isthe spiral shown in Fig. 6:86. inf isi} o=0 Section 6.11 Obtaining a Pole-Z ‘ero Model from Frequeney-Response Data 459. Let us examine the shape of the spiral in more detail, We pick a Nyquist path with a small detour to the right of the origin. The effect of the pole at the origin is the large ate at infinity with a 180° sweep, as shown in Fig, 6.85. From Eq, (6.76), for small values of « > 0, the real part of the frequency response is close to —1 because sine = w and Re[G(Jo}] © —1. Similarly. for small values of @ > 0, cos = 1 and Im|G(jo)] = ~I/e. that is, very large. negative values as shown in Fig. 6.86. To obtain the erossover points on the real axis, we set the imaginary part equal to zero Se 0. (6.17) “The solution is then = Gattis 0.12, (678) After substituting Eq, (6.78) back into Eq, (6.76), we find Ginn n=0,1,2, So the first crossover of the negative real axis is at —2/-r corresponding to n = 0. Tae first crossover of the positive real axis occurs for n = | and is located at 23x. As we cam infer from Fig. 6 86, there are an infinite number of other crossings of the real axis Finally, for « = o0, the Nyquist plot converges to the origin, Note that the Nyquist plot for « = 0 isthe mirror image of the one for « 0 ‘The number of poles in the RHP is zero (P = 0), £0 for closed loop stability we need Z = N= 0. Therefore, the Nyquist plot cannot be allowed to encircle the —1/K point. Ic will not do so as long as (6.79) 6.11 Obtaining a Pole-Zero Model from Frequency-Response Data As we pointed out earlier, itis relatively easy to obtain the frequency response of asystem experimentally. Sometimes itis desirable to obtain an approximate model, in terms of a transfer function, directly from the frequency response. The derivation of such a model can be done to various degrees of accuracy. ‘The method described in this section is usually adequate and is widely used in practice. ‘There are two ways to obtain a model from frequeney-response data. In the first case we can introduce a sinusoidal input, measure the gain (logarithm of the amplitude ratio of output to input) and the phase difference between output and input, and accept the curves plotted from this data as the model. Using the methods given in previous sections, we can derive the design directly 460 Chapter 6 ‘The Frequeney-Response Design Method EXAMPLE 6.27 Figure 6.87 {An experimental frequency response from this information, In the second case, we wish to usc the frequency data to verify a mathematical mode! obtained by other means. To do so we need to extract an approximate transfer function from the plots, again by fitting straight lines to the data, estimating break points (that is, finding the poles and zeros), and using Fig. 6.3 to estimate the damping ratios of complex factors from the frequency overshoot. The next example illustrates the second case. Transfer Function from Measured Frequency Response Determine a transfer function from the frequeney response plotted in Fig. 6.87, where frequency f i plotted in hertz L L | « | |__| Magaitde with | | hf seconde tem Asemoved ab Magnite 16 4-20 7 oo! + oo 0 Phase, £6 19 Wel tom measured Section 6.11 Obtaining a Pole-Zero Model from Frequency-Response Data 461 Solution, Drawing an asymptote to the final slope of -2 (or —40 db per decade), we assume a break point at the frequency where the phase is 90°. This occurs at fi © 1.66 Hz (ay = 2x f; = 10.4 radisee). We need to know the damping ratio in order io subtract out this second-order pole. For this the phase curve may be of more help. Because the phase around the break-point frequency is symmetric, we draw line at the slope of the phase curve at fj to find that the phase asymptote intersects the 0° line at fy = 0.71 Hz (or 4.46 radisee). This corresponds to fi/fo = 2.34, which in time corresponds to ¢ % 0.5, as seen on the normalized response curves in Fig. 63(b). The ‘magnitude curve with the second-order factor taken out shows an asymptotic amplitude gain of about 6.0 db, or a tactor of 10°" = 2.0, Because this isa gain rise, it occurs because ofa lead compensation of the form slat sbi where b/a = 2.0. If we remove the second-order terms in the phase curve, we obtain a phase curve with a maximum phase of about 20°, which also corresponds oa frequency Separation of about 2. To locate the center of the ead compensation, we must estimate the point of maximum phase based on the lead teem alone, which occurs atthe seometic ‘mean of the two break-point frequencies. The lead center seems tooccur at fa = 0.3 Hz (or a2 = 1.88 radiseo), Thus we have the relations ab(.88)? = 358, from which we can solve (ur final model is given by . 5/1.33)+ Gey = Jl = (680) (WR w+ God FT] ‘The actual data were plotted from Ge) = —___)#1 ISOs Ts AOE CA + ‘As can be seen, we found the second-order term quite easily. but the location of the lead compensation is off in center frequeney by a factor of 4/2.66 = 1.5. However, the subtraction of the second-order term from the composite curve was not done with reat accuracy, rather, by reading the curves. Again, as with the transient response, we conclude that by a bit of approximate plotting we can obtain a crude model (usually Within a factor of 1.4 (3 db) in amplitude and -£10° in phase) that can be used for control design 462. Chapter 6 The Frequency-Response Design Method Refinements on these techniques with computer aids are rather obvious, and an interactive program for removing standard first-and second-order terms. ‘and accurately plotting the residual function would greatly improve the speed and accuracy of the process. It is also common to have computer tools that can find the parameters of an assumed model structure by minimizing the sum of squares of the difference between the model's frequency response and the experimental frequency response. SUMMARY «« The frequency-response Bode plot is a graph of the transfer function mag- nitude in logarithmic scale and the phase in linear scale versus frequency in logarithmic scale, For a transfer function G(s). A=|6G0) = 1G) a6 Vike[G ony? + m[G ion? sPPMIEGON] — Gey # For a transfer function in Bode form, soy = x, lott Dion +1 tw) = Kylee ten the Bode frequency response ean be easily plotted by hand using the rules described in Section 6.1 «Bod plot can be obtained using computer algorithms (bode in Maras), but hand-ploting skills are stil extremely helpful «For a second-order system the peak magnitude of the Bode plots related tothe damping by 1 Go| = = 1GUe)l = 5 ate = oy «A method of determining the stability of a closed-loop system based on response of the system’s open-loop transfer function is the [Nyquist stability criterion. Rules for plotting the Nyquist plot are described ection 6.3. The number of RHP closed-loop roots, Z. is given by =N+P, where NV = number of clockwise encirclements of the ~ 1 point, P = number of open — loop poles in the RHP. Figure 6.88 Typical system Summary 463 The Nyquist plot may be obtained using computer algorithms (nyqui st in Maran), ‘The gain margin (GM) and phase margin (PM), can be determined directly by inspecting the open-loop Bode plot or the Nyquist plot. Also, use of Martas’s margin function determines the values directly. For a standard second-order syster damping by Eq. (6.32) . the PM is related to the closed-loop =m ~ 100 The bandwidth of the system isa measure of speed of response, For control systems it is defined as the frequency corresponding to 0.707 (3 db) in the closed-loop magnitude Bode plot and is approximately given by the crossover frequency, w:, which is the frequency at which the open-loop gain curve crosses magnitude 1 g The vector margin is a single-parameter stability margin based on the clos- est point of the Nyquist plot to the critical point —1/K Forastable minimum-phase system, Bode’s gain-phase relationship uniquely relates the phase tothe gain of the system and isapproximated by Eq, (6.33) LG (jo) = n x 90° where 1 is the slope of |G ja) in units of decade of amplitude per decade of frequency. The relationship shows that, in most cases, stability is ensured if the gain plot crosses the magnitude I fine with a slope of —1 Experimental frequency response data of the open-loop system can be used directly for analysis and design of a closed-loop control system with no analytical model, For the system shown in Fig. 6.88, the ope response of GD, Ts joop Bode plot is the frequency the closed-loop frequency response is obtained from GD/A+GD), ‘The frequency response characteristics of several types of compensation have been described, and examples of design using these characteristics hhave been discussed. Design procedures were given for lead and lag com pensators in Section 6.7. The examples in that section show the ease of selecting specific values of design variables, a result of using frequency response methods. A summary was provided at the end of Section 6.7.5 on page 440, @4HeH~ | 464 Chapter 6 The Frequency-Response Design Method «Lead compensation, given by Eq. (6.38), Ts+1 DE) =F act Ts+1 is a high-pass filter and approximates PD control. It is used whenever substantial improvement in damping of the system is required. It tends to increase the speed of response of a system for a fixed low frequency gain, ‘+ Lag compensation, given by Eq. (6.51), Ts+1 DO) =a (681) isalow-pass filter and approximates Pl control. Itis usually used to increase the low-frequency gain of the system so as to improve steady-state response for fixed bandwidth. For a fixed low-frequency gain, it will decrease the speed of response of a system, «The Nichols plot isan alternate representation of the frequency response as a plot of gain versus phase and is parameterized asa function of frequency. + Tracking-error reduction and disturbance rejection can be specified in terms of the low-frequency gain of the Bode plot. Sensor noise rejection can be specified in terms of high-frequency attenuation of the Bode plot (see Fig. 6.72). ‘* Time delay can be analyzed directly by sketching the Nyquist plot. ‘+ Transfer-function poles and zeros can be estimated from the measured frequency response. Review Questions 1. Why did Bode suggest plotting the magnitude of a frequency response on log-log coordinates? 2. Define a decibel. ‘3. What i the transfer function magnitude ifthe gain is listed as 14 db? 4, Define gain crossover. '5. Define phase crossover. 6, Define phase margin, PM, 7. Define gain margin, GM. 8. What Bode plot characteristic s the best indicator ofthe closed-loop step response overshoot? 9. What Bode plot characteristic is the best indicator ofthe closed-loop step response 10. n 6a. Problems 465 A certain control system is required to follow sinusoids which may be any frequency inthe range0 « wy = 450 radisecand have amplitudes up to units with (sinusoidal) steady-state error tobe never morethan 0.01, Sketch (or describe) the corresponding Performance function W% (0) ‘What isthe principal effect of a lead compensation on Bode plot performance mea ‘What isthe principal etfect of a lag compensation on Bode plot performance mea How do you find the K, of a type I system from its Bode plot? l. Why do we need to know before hand the number of open-loop unstable poles in ‘order to tell stability from the Nyquist plot? Whats the main advantage in control design of counting the encitclements of =1/K of D(ja)GCJoo rather than et circlements of —1/K of D{jeG( jw)? Define a conditionally stable feedback system, How can you identify one on & Bode plot? Problems Problems for Section 6.1 (a) Show that ay in Eg, (62) is given by = [0 J] = -wocti0 =P. and a= [ow 4°] r+ 70 tr By assuming the output can be writien as N11) = aye ae derive Eqs (64)-(65). wy Caleutate the magnitude and phase of t Gis m0 by hand for w= 1,2.5, 10,20, 50, and 100 radlsee (by Sketch the asymptotes for G(s) according tothe Bode plot rules, and compare these with your computed results from part (a) 466 Chapter 6 ‘The Frequency-Response Design Method 63. 64, 65. Sketch the asymptotes of the Bode plot magnitude and phase for each of the following open-loop transfer functions. After completing the hand sketches verify your result using MATLAB. Turn in your hand sketches and the MATLAB results, ‘on the same scales. 4000, (2) Lis) = 100. ©) 1) = coe OSE 1 © LO Se HOMEED : 1 MO= eae 1We+4) Los) = tt = Geeta 5 10406 +0.) SCH DTH RS OD (+ 5y(s-+3) Sree +H 4sts +10) CFO +5 © © Liy= @ Li) = hy) Lis) = O19 = THI Fs +50, Real poles and zeros. Sketch the asymptotes of the Bode plot magnitude and phase for each of the following open-loop transfer functions. After completing, the hand sketches verify your result using MATLAB, Turn in your hand sketches tnd the MATLAt results on the same scales, 1 ©) LO) = aye F (+2) INET ITT = +2 +6 (9 LO) = EHO FSG FT cH +20 +4) @ L0)= eS Het eI : (b) Lis) = Complex potes and zeros. Sketch the asymptotes ofthe Bode plot magnitude and phase for each of the following open-loop transfer functions and approximate the transition atthe second order break point based on the value of the damping ratio. Alter completing the hand sketches verify your esult using MATLAB. Tura in your hand sketches and the MATLAH results on the same scales 1 (0 0 SEE fe) Lis) = sh © Liss = sto Problems 467 66. Muliple poles a the origin. Sketch the asymptotes of the Bode plot magnitude 61. and phase foreach ofthe following open-looptranster functions. After completing the hand sketches verify your result using MATLAB. Turn in your hand sketches and the MATLAB results on the same seales. © w= 4 Des S644) y= een OO rea, Mixed real and complex poles. Sketch the asymptotes ofthe Bode plot magnitude and phase for each of the following open-loop transfer functions. Embellish the asymptote plots with a rough estimate of the transitions for each break point After compicting the hand sketches verify your result using MATLAB. Tun in your haul sketches and the MATLAB results on the same scales +2) (a) L00) = ‘ MO) ina +a gr erase) OO Netw TB © Li =~ * +35) (@) Lis) = E42? pe +68) Foe ye a $85) fw Ly =f Ft 2G 3) Aight al plane pote and zeros ksh te asymptote of he Bo pot tude and phase for each of he lowing ope oop taser one Makes the phase asymptote properly ake he RUPsinglaiy mosesoon oy aeeeins "he Complex plane to sce hw the Ls) changes ns ges rom to is ey completing the hand sketches verity yours using IATL AD Torn vor nnd sfctohes nd the MATLaw oslo the ame ele (8) Li) = SGP 1 the model for acase of magnetic vitaton with ead compensation, 8) = pit? 1 the magnetic levi es (©) L0) = SF fay gar ap! the magnetic levitation systom with integral con- {rol and lead compensation (©) Lo) = 551 a) Lis 52041 eo) S68 + 20)"(s" — 2s 468 Chapter 6 ‘The Frequency-Response Design Method Figure 6.89 Magnitude portion of Bode plot for Problem 6:9 @) Li) = 642 Oe sis = Ds +6" © Lis 1 ole S43) 69. Acertainsystem isrepresented by the asymptotic Bode diagram shown in Fig. 6.89, Find and sketch the response of this system to a unit step input (assuming zero initial conditions). oy — Ww 1001000 6.19, Prove that a magnitude slope of ~1 in a Bode plot corresponds to ~20 db per decade or ~6 db per octave 6.1. A normalized seeand-orcorsystem witha damping ratio ¢ zero is given by S andun additional slat) Peel Gu) = ‘Use MATLAB to compare the My from the step response of the system for a = 0.01,0.1, 1, 10, and 100 withthe IM, from the frequency response of each ease. Is there a correlation between M, und M,? 6412, A normalized second-order system with ¢ by .$ and an additional pole i given : 1 OO = ipa tle ese Draw Bode plots with p =0.01, 0.1, 1, 10and 100. What conclusionscan you draw about the effect of an extra pole of the bandwidth compared to the bandwidth for the second-order system with no extra pole? 6.13, For the closed-loop transfer function T= FT haa re derive the following expression for the bandwidth ony of T(s) in terms of ay, and aeyt— cow = ony [Assuming oy = 1, plot ogy for 0 ¢ <1 Problems 469 6.14. Consider the system whose transfer funetion is This is a model of a tuned circuit with quality factor Q. (a) Compute the magnitude and phase of the transfer function analytically, and plot them for Q = 0:5, 1,2, and 5 as a function of the normalized frequency (b) Define the bandwidth as the distance between the frequencies on ether side of ap where the magnitude drops to 3 db below its value at « and show that the bandwidth is given by owe: (3) (©) What i the elation between Q and ¢? ADC voltmeter schematic is shown in Fig. 6.90. The pointer is damped so that its maximum overshoot toa step input is 109 (a) What isthe undamped natural frequency of the system’? (b) Whats the damped natural frequeney of the system? (©) Plot the frequency response using MATLAB fo determine what input fre- quency will produce the largest magnitude output? (@) Suppose this meter is now used to measure a 1-V AC input witha frequency ff 2radisee. What amplitude will the meter indicate alter initial transients have died out? What isthe phase lag of the output with respect tothe input? Use a Bode plot analysis to answer these questions. Use the Isim command in MATLAB to verify your answer in part (A) Figure 6:90 2 Voltmeter schematic a 1402 10° kg me KA 10% kg mb 1 inp torqe = v= input voltage k 470 Chapter 6 ‘The Frequency-Response Design Method 6.16, 617. 619. Problems for Section 6.2 Determine the range of K for which each of the following systems is stable by making a Bode plot for K = 1 and imagining the magnitude plot sliding up oF down until instability results. Verify your answers using a very rough sketch of a root-locus plot. Kw +2) pat O) KG = nee KG +10 +1) (+ 1005 +5)" @) KGw)= 1 KGW) = Determine the range of K for which each of the following systems is stable by ‘making a Bode plot for K = 1 and imagining the magnitude plot sliding up or ddown until instability eesults. Verify your answers using a very rough sketeh of a root locus plot. cou) = K+) (a) KG) = REED = K+) ) Koi) = SEHD. x ©) G(s) = : Aa wae Kies? to) KG) = SEER Problems for Section 6.3 (a) Sketch the Nyquist plot for an open-loop system with transfer function 1/2; that is sketch t \where C is contour enclosing the entire RHP, as shown in Fig, 6.17. (Hint: ‘Assume (Cy takes a small detour around the poles at ¢ = 0, as shown in Fig 6.27) (b) Repeat part (a) for an open-loop system whose transfer function is G(s) 1? +0) ‘Sketch the Nyquist plot based on the Bode plots for each of the following sys- tems, then compare your result with that obtained using the MATLAB command. nyquist: Ko+D () KG) = READ &) KG) Kk oy = Kiet 10-41) (e+ 100,66 +2)" (@) Using your plots, estimate the range of K for which each system is stable, and qualitatively verify your result using a rough sketeh ofa root-locus plot (©) KGW) Figure 6.91 Control system for Problem 6.21 Problems 471 6.20, Draw a Nyquist plot for Draw the Nyquist plot for the system in Fig. 691. Using the Nyquist stability criterion, determine the range of K for which the system is stable, Consider both positive and negative values of K 622. (a) Fore 1 to 100 radisee, sketch the phase of the minimum-phase system set Gy = 41 (= S510 and the nonminimum-phase system noting that cj ~ 1) decreases with «rather than increasing () Does a RHP zero atfect the relationship between the ~1 enctclements on a polar plot and the number of unstable closed-loop roots in Ea. (628)? (6) Sketch the phase ofthe following unstable system for w = 0.1 to 100 rade: Gis) = (@) Check the stability ofthe systems in (a) and (c) using the Nyquist eriterion on KG(s). Determine the range of K for which the closed-loop system is stable, and cheek your results qualitatively using a rough root-locus sketch. Problems for Section 6.4 623, The Nyquist plot for some actual control systems resembles the one shown in Fig 692. What are the gain and phase margin(s) forthe system of Fig. 6.92 given that @ = 0.4. f= 13, and ¢ = 40°. Describe what happens to the stability ff the system as the gain goes from zero to a very large value. Sketch what the corresponding root locus must look lke for such a system. Also sketch what the corresponding Bode plots would look like forthe system. 472. Chapter 6 Figure 6.92 Nyquist plot for Problem 6.23 ‘The Frequeney-Response Design Method 624, 6.26, 4 info} q The Bode plot for 100f(5/10) +1 G9) = Ty) = 1/100) +1] {s shown in Fig. 6.93 (a) Why docs the phase start at 270° at the Tow frequencies? (b) Sketch the Nyquist plot for G(s) (©) Is the closed-loop system derived from Fig, 693 stable? (@) Will the system be stable if the gain is lowered by a factor of 100? Make a rough sketch of a root locus for the system and qualitatively confirm your Suppose that in Fig. 6.94, ‘ 2518+) Gs) = — TEED FISH Use MATLAB's margin fo calculate the PM and GM for G(s) and, based on the Bode plots, conclude which margin would provide more useful information tothe ‘control designer for this system, Consider the system given in Fig. 695, (a) Use MATLAB to obtain Bode plots for K = 1 and use the plots to estimate the range of K for which the system will be stable. (b) Verify the stable range of K by using margin to determine PM for selected values of K (©) Use rlocus and rloctind to determine the values of Kat the stability bound (@) Sketch the Nyquist plot of the system, and use it to verify the number of ‘unstable roots for the unstable ranges of K (€) Using Routh’s criterion, determine the ranges of K for closed-loop stability of this system. Figure 6.93 Bode plot for Problem 6.24 Figure 6.94 Control system for Problem 6:25 Figure 6.95 Contra system for Problem 6.26 Problems 473 1000 © 100 0 0 2 1 +0 a ‘ 20 oor “0 ont o bor of 1 10 100 1000 satvadhec) xr ve — om 011 01m 1000 stead) ais) y Ln 6.27, Suppose that in Fig, 6.94, ees 26 ra OOS Fat +027 16) Use MATLAB's margin to calculate the PM and GM for G(s) and comment on ‘whether you think this system will have well damped closed-loop roots, 474 Chapter 6 6.28, 629, 6.0, 631. The Frequency-Response Design Method For a given system, show that the ultimate period and the corresponding ul- timate gain K, for the Zeigler-Nichols method can be found using the following: (a) Nyquist diagram. (b) Bode plot (@) Root locus Ifa system has the open-loop transfer function OO = 64 Rand with unity feedback, then the closed-loop transfer function is given by Tis) = Bays Verify the values of the PM shown in Fig. 6.36 for ¢ = 0.1, 04, and 07. Consider the unity feedback system with the open-loop transfer function K Gs) = —_* (9 SPD + OAC +11 (a) Use MATLAR to draw the Bode plots for Glo) assuming K = L (b) What gain Kis required for a PM of 45°? What is the GM for this value of a (©) What is K, when the gain X is set for PM (@) Create a root locus with respect to K and indicate the roots for a PM of 45 4s? For the system depicted in Fig, 6.96(a), the transfer-function blocks are defined by 1 L Ge)= and His) = Fo) 7 a (a) Using locus and rlocfind, determine the value of K atthe stability boundary. (b) Using docus and rocfind, determine the value of K that will produce roots with damping corresponding to ¢ = 0.707. Whats the gain margin of the system ifthe gain is set to the value determined in part (b)? Answer this question without using any frequeney response methods @ Create the Bode plots for the system, and determine the gain margin that resulls for PM = 65-, What damping ratio would you expect for this PM? (©) Sketch a root locus for the system shown in Fig, 6.96(b). How does it differ ‘rom the one in part (a)? (D For the systems in Figs 6.96(a) and (b), how does the transfer function ¥a(s)/R(s) differ from ¥,(s}/R(s)? Would you expect the step response to ‘(1 be different for the two cases? Problems 475 & : eHeHel not) =" Me) Figure 6.96 Black ciagram for Problem 6:31. (a) unity feedback; (b) 4s) in feedback. Figure 6.97 Control system for Problem 6.32 Magnetic tape-drive speed contol 632, For the system shown in Fig. 6.97, use Bode and root-locus plots to determine the gain and frequency at which instability occurs. What gain (or gains) gives a PM ‘of 20°? What isthe gain margin when PM = 20°? 633. A magnetic tape-drive speed-control system is shown in Fig. 698. The speed ‘sensor isslow enough that its dynamics must be included. Thespeed-measurement time constant is ty = 0.5 sec; the reel time constant is t, = J/b = 4 see, where b= the output shaft damping constant = I N.m-sec; and the motor time constant (a) Determine the gain K’ required to keep the steady-state speed error to less than 7% of the reference-speed setting (b) Determine the gain and phase margins of the system. Is this a good system design? ple eZ snd motor penis ee x | Tove roKE py AS oo 634, For the system in Fig, 6.99, determi criterion (a) to determine the range of values of K’ (positive and negative) for which the system will be stable and (by todetermine the number of roots in the RHP for those values of K for which the system is unstable. Check your answer using a rough root-locus sketch, the Nyquist plot and apply the Nyquist 476 Chapter 6 Figure 6.99 Control system for Problems 6.34, 6 61, and 662 Figure 6.100 Control system for Problem 6.35 Figure 6.101 Control system for Problem 6.36 no) ET LAI 638, ko The Frequency-Response Design Method LEE i ‘Semsor - For the system shown in Fig. 6.100, determine the Nyquist plot and apply the Nyquist eriterion (a) todetermine the range of values of K (positive and negative) for which the system willbe stable and (b) to determine the number of rots in the RHP for those values of K for which the system is unstable. Check your answer using a rough root-locus sketch, 6.36, OCHEH For the system shown in Fig. 6.101, determine the Nyquist plot and apply the Nyquist criterion (a) to determine the range of values of K (positive and negative) for which the system willbe stable and (b) to determine the number of roots in the RHP for those values of K for which the system is unstable. Check your answer using a rough root-locus sketch, 637. ‘The Nyquist diagrams for two stable, open-loop systems are sketched in Fig. 6.102. The proposed operating gain is indicated as Ky, and arrows indicate increasing frequency. In each case give a rough estimate ofthe following quantities for the closed-loop (unity feedback) system (a) Phase margin, ¢b) Damping rato, (€) Range of gain for stability (if any). (a) System type (0.1.0 2). Figure 6.102 Nyquist plots for Problem 6.37 638. 640, 477 Problems @ » The steering dynamics of a ship are represented by the transfer function K(-15/0.142) +1) UO) ay = SEpOaas + HOMO BG) +) where » is the ship's lateral velocity in meters per second, and 4, is the rudder angle in radians. (a) Use the MATLAB command bode (0 plot the log magnitude and phase of G(jw) for K = 02. (b) On your plot, indicate the crossover frequency. PM, and GM. (6) Is the ship steering system stable with K = 0.2? (@) What value of K would yield a PM of 30° and what would the crossover frequency be? 1. For the open-loop system Kt) KOO) = 2 TOF tcemine the value for Kat the stability boundary and the values of Kat the points where PM = 307 Problems for Section 6.5 The frequency response of a plant in a unity feedback configuration is sketched in Fig. 6.103. Assume the plant is open-loop stable and rainimum phase. (2) What isthe velocity constant K, for the system as drawn’? (b) What isthe damping ratio of the complex poles at «= 101"? (€), What isthe PM of the system as drawn’? (Estimate to within £10") 478. Chapter 6 The Frequency-Response Design Method Figure 6.103 Magnitude trequency response for Problem 6.40 @ 20 ie —| 20 om siti dg 1 W200 stead) GAL. For the system 643, 100(s/a +1) 56+ G+ where b = 10a, find the approximate value of a that will yield the best PM by sketching only candidate values of the frequency response magnitude, as Problem for Section 6.6 . For the open-loop system Keb, KOO) = SEF TOF determine the value for K that will yield PM > 30° and the maximum possible closed-loop bandwidth, Use MATLAB to find the bandwidth, Problems for Section 6.7 For the lead compensator tan“(To) — tana, (b) Show thatthe frequeney where the phase is maximum is given by Figure 6.104 Control system for Problem 6.45 Problems 479 ‘and that the maximum phase corresponds to Fa (€) Rewrite your expression for wings to show thatthe maximum-phase frequency ‘curs a the geometric mean of the Wwo corner frequencies on a logarithmic stale ita 1) logon: = $ (log # + log A ) (a) To derive the same results in terms of the pole-zer0 locations, rewrite Dis) Dis) = SE, rap ‘and then show thatthe phase i given by o-oer(g)-me (i such that one ~ vill ence the frequency at which the phase is maximum is the square root of the product of the pole and zero locations, 6.044, For the third-order servo system 6s, 50,000 69 = FO +5 Design a lead compensator so that PM 250° and yy > 20 radisee using Bode plot sketches then verify and refine your design using MATLAB, For the system shown in Fig. 6.104, suppose that a a 58S eS DeSeD Design a lead compensation D(s) with unity DC gain so that PM > 40° using Bode plot sketches, then verify and refine your design using MATLAB. Whats the approximate bandwidth of the system? 480. Chapter 6 6.46, 67. 69, The Frequency-Response Design Method Derive the transfer function from 7; {0 6 for the system in Fig 6.68. Then apply the Final Value Theorem (assuming T, = constant) to determine whether 8(20) is nonzero for the following two cases: (a) When D(s) has no integral term, where lim, -0 D(s) = constant. (6) When Dis) hasan integral term: 1) =P, where fim, .9 D'(s) = constant The inverted pendulum hasa transfer function given by Eq. (2.35), whichissimiliar to ‘ 1 ow = 55 (a) Design a lead compensator to achieve a PM of 30° using Bode plot sketches, then verify and refine your design using MATLAB (b) Sketch a root locus and correlate it with the Bode plot ofthe system, (©) Could you obtain the frequency response of this system experimentally? ‘The open-loop transfer function of a unity feedback system is K 6) = ——* ___ (= GRE DGROFD (a) Design a lag compensator for G(s) using Bode plot sketches so that the closed-loop system satisfies the following specifications: i. The steady-state error to a unit ramp reference input is less than 0.01 i. PM 2407 (b) Verify and refine your design using MATLAB, ‘The open-loop transfer function of a unity feedback system is K OO) = GT epMe (2) Design a lead compensator for Gis) using Bode plot sketches so that the closed-loop system satisfies the following specifications: i. The steady-state error to a unit ramp reference input is less than 0.0 fi, For the dominant closed-loop poles the damping ratio ¢ = 0.4. (b) Verify and refine your design using MATLAB including a direct computation ‘of the damping of the dominant closed-loop poles 1. ADC motor with negligible armature inductance i tobe used in a position control system. Its open-loop transfer function is given by 50 OO RD (@) Design a compensator for the motor using Bode plot sketches so that the closed-loop system satisfies the following specifications 481 Problems 4. The steady-state error to a unit ramp input is less than 1/200, fi, The unit step response has an overshoot of less than 2 fii, The bandwidth of the compensated system is no Tess than that of the ‘uncompensated system, (b) Verily andior refine your design using MATL including a direct computa: tion of the step response overshoot 651. ‘The open-loop transfer function of a unity feedback system is K OO) = BM +s (a) Sketch the system block diagram including input reference commands and (b) Design a compensator for G(s) using Bode plot sketches so thatthe closed- Joop system satisfies the following specifications: 4. The steady-state error toa unit ramp input is less than 0.01 fi, PM 45 li, The steady-state error for sinusoidal inputs with « < 0.2 radise than 1250. iv, Noise components introduced with the sensor signal at frequencies preater than 200 radisec are to be attenuated at the output by atleast a factor of 100, (©) Verily andior refine your design using MATLAB including a computation of the closed-loop frequeney response to verity (iv). 652. Consider a type I unity feedback system with OO = aD Design a lead compensator using Bode plot sketches so that K, = 20sec and PM > 40), Use MATLAS to verify and/or refine your design so that it mec specifications 653. Consider a satelite attitude control system with the transfer function 0.0565 +25) OO = FeO +H Amplitude stabilize the system using lead compensation so that GM > 2 (6 db). | ‘and PM = 45’, keeping the bandwidth as igh as possible witha single lead, none mode of operation the autopilot ofa jt transport isused to control altitude. For the purpose of designing the altitude portion of the autopilot loop. only the long-period airplane dynamicsare important. The linearized relationship between altitude and elevator angle forthe long-period dynamics is Ms) aa 201s +0001) fry see WHF O0Is + 0.0025) deg Gs) 482. Chapter 6 Figure 6.105 Control system for Problem 6.54 ‘The Frequency-Response Design Method Lad te LAST # The autopilot receives from the altimeter an electrical signal proportional to alt tude. This signal is compared with a command signal (proportional tothe altitude selected by the pilot), and the difference provides an error signal. The error sig nal is processed through compensation, and the result is used to command the clevator actuators. A block diagram of this system is shown in Fig, 6.105. You have been given the task of designing the compensation. Begin by considering a proportional contrat law D(s) = K. (a) Use MATLAB to draw a Bode plot of the open-loop system for D(s) = K = 1 (b) What value of K would provide a crossover frequency (i.e. where |G] = 1) of 0.16 radisec? (©) For this value of K, would the system be stable i the loop were closed? (@) What is the PM for this value of K? (€) Sketch the Nyquist plot of the system, and locate carefully any points where the phase angle is 180° or the magnitude is unity. (Use MATLAB to plot the root locus with respect to K’, and locate the roots for your value of K from part (b). (@) What steady.state error would rerult if the command was a step change in altitude of 1000 1? For parts (hand (i), assume a compensator of the form Ts) dw=w re y Choose the parameters K. 7. and so that the crossover frequency is (0.16 radisee and the PM is greater that 50°. Verify your design by super imposing a Bode plot of D(s)G(s)/K on top of the Bode plot you obtained for part (a), and measure the PM directly. (Use Maran to lot the root locus with respect to K for the system including the compensator you designed in part (h). Locate the roots for your value of from part (h), (Altitude autopilotsalso have a mode where the rate of climbs sensed directly and commanded by the pilot. J. Sketch the block diagram for this mode, fi, Define the pertinent G(s) iil, Design D(s) so that the system has the same erossover frequency as the altitude hold mode and the PM is greater than 50 655. Fora system with open-loop transfer function 0 Ge) = = Tena C+ design a lag compensator with unity DC gain so that PM 240°. What is the approximate bandwidth of this system? 656, 658, Problems 483 For the ship-tecring system in Problem 6.38, (a) Design a compensator that meets the following specifications: 4. Velocity constant K, =2 Hi PM = 50 fii, Unconditional stability (PM. > 0 for all quency). (b) For your final design, draw a root locus with respect to K', and indicate the location of the closed-loop poles. ‘othe erossover fre- . For a unity feedback system with Gis) = ——__} ___ (0s) (& = 4055 (+) (G++) (4) A lead compensator is introduced with a = 1/5 and a zero at 1/T = 20. How must the gain he changed to obtain crossover at oj, = 31.6 radisec, and what isthe resulting value of K,? (b) With the lead compensator in place, what isthe requited value of K for alag compensator that will readjust the gain to the original Ky value of 1007 (©) Place the pole ofthe lag compensator at 3.16 rade, and determine the 2ero Jocation that will maintain the crossover Irequency at «. = 31.6 rade. Plot the compensated frequeney response on the same graph, (a) Determine the PM of the compensated design Golden Nugget Airlines had great success with their free bar near the tail of the airplane (see Problem 5.41), However, when they purchased a much larger airplane to handle the passenger demand, they discovered that there was some Alexiiliy in the fuselage that caused a lot of unpleasant yawing motion atthe rear of the aiplane when in turbulence anid was causing the revelers tospill their drinks ‘The approximate transfer function for the dutch roll mode (see Section 93.1) is, rs) ___8.75¢4s? +048 +1) 5) ~ Goi + he +02% FD where ris the airplane's yaw rate and 5, isthe rudder angle. In performing finite element analysis (FEA) of the fuselage structure and adding those dynamics t0 the dutch roll motion, they found that the transfer function needed additional terms that reflecied the fuselage lateral bending that accursed due to excitation from the rudder and turbulence. The revised transfer function is His) ___875UH 50)” GOL Dee + 024+ ( as) 1 where wy isthe frequency of the bending mode (= 10 rad/sec) and ¢ isthe bending ‘mode damping ratio (= 0.(2), Most swept wing airplanes have a "yaw damper ‘which essentially feeds back yaw rate measured by a rate gyro to the rudder ‘with «simple proportional control law. For the new Golden Nugget airplane the proportional feedback gain. &’,is 1. where ts) = KG) (st) 484° Chapter 6. The Frequen y-Response Design Method (a) Moke # Bode plot of the open-loop system, determine the PM and GM for the nominal design, and plot the step response and Bode magnitude of the closed-loop system. What isthe frequency of the lightly damped mode that is causing the difficulty? (b) Investigate remedies to quiet down the oscillations, but maintain the same Jow-frequeney gain in order not to affect the quality ofthe dutch roll damping provided by the yaw rate feedback. Specifically, investigate one ata time: J. Incteasing the damping of the bending mode from ¢ = (0.02 to ¢ = 1.04 (would require adding energy absorbing material in the fuselage structure). i. Increasing the frequency of the bending mode from a = 10 radisee 10 ws = 20 radisec (would require stronger and heavier structural ele ments). ii, Adding a low-pass filter in the feedback, that is, replace K in Eq, (6.84) with Dis) where 1 DO (685) Pick x, so that the objectionable features of the bending mode are re~ duced while maintaing the PM > 60” I, Adding a notch filter as described in Section 5.5.3. Pick the frequency fof the natch zera to be at vo with a damping of ; = 004 and pick the denominator poles to be (4/100 + 1)*, keeping the DC pain ofthe filter equal to 1 (©) Investigate the sensitivity of the two compensated designs above (ili and iv) by determining the effect of a reduction in the bending mode frequeney of 10%. Specifically, reexamine the two designs by tabulating the GM, PM, closed loop bending mode damping ratio, and resonant peak amplitude, and {qualitatively describe the differences inthe step response. (@ What do you recommend to Golden Nugget to help the customers quit spilling their drinks? (Telling them to get back in their seas is not an ac: ceptable answer for this problem! Make the recommendation in terms of provements to the yaw damper.) Problems for Section 6.8 659, A feedback control system is shown in Fig. 6.106. The closed-loop system is specified to have an overshoot of less than 30% to a step input (a) Determine the corresponding PM specification inthe frequency domain and the corresponding closed-loop resonant peak value M, (see Fig. 6.37) (b) From Bode plots of the system, determine the maximum value of K that satisfies the PM specification, Figure 6.106 Control system for Problem 6.59 Problems 485 (©) Plot the data from the Bode plots {adjusted by the obtained in part (b)] on a copy of the Nichols chat in Fig 6.73 and determine the resonant peak mag- nitude M,. Compare that with the approximate value obtained in part (a) (a) Use the Nichols chart to determine the resonant peak frequency «, and the closed-loop bandwidth, 6.60. The Nichols plot of an uncompensated and a compensated system are shown in Fig. 6.107 (2) What are the resonance peaks of each syster (b) What are the PM and GM of each system? (©) What are the bandwidths of each system? (@) What type of compensation is used? rcompensated Conpensied ~ Closed toon magnitude Closed oop pe rs 16 1 1a 0" wo ma wo Figure 6.107 Nichols plot for Problem 6.60 486 Chapter 6 The Frequency-Response Design Method 6464, Consider the sjem shownin Fie 698 (a) Construct an inverse Nyquist plot of [Y(je)/E(ja)]-* (8) Show how the vl of for nual sabi an Be ed itech tere qu pl (o) For k= 2a determine the ain ad phase margins (8) Contra rots poor te se, and en conesponding pins Inthe two pt To what damping ratoe¢ othe GMtand PMO part (henge 6.2, Anant plant haste want function FO 7 O=0F A simple con op isto bese around in he same maner sth Back dha ine 0 (a) Const an inverse Nya plo 0 (8) Choos alot to provid PMot 4, Whatithconesponding GM? (Wht can ou ine fm Your pa about the sab af the sem Men (# Constr root esp forthe sem, and entity coresgoning pits into og nihseneohat luca oes Mt 86 conepond? 64 Consider the stem shown in Fie 6108) (a) Cont Bode pot or thes 0) Use your Bode plot seteh an mere Nya plo (0) Comiderciesingaconro oop around Gass in F-61080), Using thease plcaaqae rend emsur Bae pan ales Rt PM eiui 200. 10.and Wha vet sls FM 3? (a) Constr ote pad lel he sane vas Kon the as To Sin ac of Sos oth prot MLM wh coespond? Compare he Nia PA winch apron te 838% Faure 6.108 ie Cant! te Y yo) EL} fea] or Pasion 803 Problems for Section 89 6.64 Conder sem ih he oerdoop ans nation oop gain 1 CO= Fee 6.65, 6.06. Figure 6.109 Contra system constraints for Problem 6.66, Magnitude, [u Problems 487 (2) Create the Bode plot forthe system, and find GM and PM. (b) Compute the sensitivity sensitivity function and plo its magnitude frequeney response. (€) Compute the vector margin (VM). Prove thatthe sensitivity function S(s) has magnitude greater than 1 inside a circle ‘with a radius of 1 centered at the —1 point, What does this imply about the shape ‘of the Nyquist plot if closed-loop control isto outperform open-loop control at all frequencies? ‘Consider the system in Fi 6.104 with the plant transfer Funetion Ge) says (a) We wish to design a compensator D(s) that satisfies the following design specifications: K, = 100, PM > 45 Ill, Sinusoidal inputs of up to I radisee to be reproduced with = 2% error Sinusoidal inputs with a frequency of greater than 108 radisee to be attenuated at the output to = 5% of their input value, (b) Create the Bode plot of Gis), choosing the open-loop gain so that Ky, = 100. (©) Show that a sufficiens condition for meeting the speeitieation on sinusoidal inputsis thatthe magnitude plot lies outside the shaded regions in Fig. 6.109, Recall that y_ Ke RO T+KG E RO TERE (a) Explain why introducing a lead network alone cannot meet the design specifications (©) Explain why a lag network alone cannot meet the design specifications. (® Develop full design using a lead-lag compensator that meets all the design specifications, without altering the previously chosen low frequency open: loop gain. 1000 | ry ™ oF i peace cites eee so roo eee mac tats) 488 Chapter 6 The Frequency-Response Design Method Problems for Section 6.10 6.67. Assume thatthe system a= hus 20.2-see time delay (7 =0.2 sec). While maintaining a phase margin = 40 Find the maximum possible bandwidth using the following: (a) One lead-compensator section Dis) = Ke where bja = 100; (b) Two lead-compensator sections w= « (44) sb, where bja = 10, (©) Comment on the statement inthe text about the limitations on the bandwidth, imposed by a delay. 668, Determine the range of K for which the following systems are stable (@) Gis) ) Ge) se FI 6.69. In Chapter 5, we used various approximations forthe time delay. one of which is the first-order Padé Lens Ts? Using frequency response methods, the exact time delay Hy) = ‘can be used. Plot the phase of (5) and Hs(s) and diseuss the implications. 6.70, Consider the heat exchanger of Example 2.17 with the open-loop transfer function Gs) = Tos er (a) Design a lead compensator that yields PM = 45° and the maximum possible closed-loop bandwidth, (by Design & PI compensator that yields PM > 45° and the maximum possible closed-loop bandwidth, Problems 489 Problems for Section 6.11 6.71. The Bode plot in Fig. 6.110 is for a transfer funetion of the form ks aehoat he Gor= where K, Tj, and Ts are positive constants, Determine values for these three constants from the Bode plot Figure 6.110 + 1 Bode pot for Problem 6.71 | 30 1090 = & 100 4 1040 100 400 1000 wtradsec) 0" Tos 124 Wa 100400 1000 6.7, You are given the experimentally determined Bode plot shown in Fig. 6.111. De- sign @ compensation that wil yield a crossover frequency of w, = 10 radi/see with PM = 75 490. Chapter 6 The Frequency-Response Design Method Figure 6.111 Bode plat for Problem 6.72 190 100 o4 aor oon 00001 L—_— or or =180"| Phe, £6 om 360" 4 6.73, Consid 110 100 }ro000 sot) T1010) 10000 ‘w(rades) the frequeney-response plot in Fig, 6.112, which was obtained from a finite-element analysis Determine a transfer function that approximately matches. the measu red frequency response up to 60) Hz, 10 0 200 © 0 » aur 1 oo g e 2 -o | 00" o1 20 1000" | ot 40 100 pit tap LL 12 46 10 20 4060100 200 400 1000 124 6 19 20 aan 100 200 400 1000 Fate pity ® ® Figure 6.112. Frequency-response plot for Problem 6.73 TABLE 64 Problems 491 6.74. The frequency response data shows in Table 6.1 were taken froma DC motor that 's to be used in a position control system. Assume that the motor is Hinear and sinimum-phase. (a) Estimate G(s), the transfer function of the system, (b) Design a series compensator for the motor so that the closed-loop system ‘meets the following speeitieations: ‘The steady-state error to a unit ramp input i less than 0.01 PM > 45 Frequency-Reponse Data for Problem 6.74 aw aaleed) [GO] (85) @ tadisec) [GG ad) _w Fadiees) ITH ee) a4 600 30 305 600 200 02 540 40 270 650 210 03 500 50 23.0 800 240 os 480 70 195 1000 © 30.0 os 420 100 140 2000 © 480 10 40.0 200 20 300.0 © 580 20 340 400-100 500.0 72.0 492 T State-Space Design i el Chapter Overview This chapter begins by considering the purposes and advantages of using] state-space design. In Section 7.2 we review the development of state- variable equations from block diagrams. We then solve for the dynamic response, using state equations for both hand and computer analysis. Having covered these preliminary fundamentals, we next proceed to the] major task of control system design via state space. The steps of th design method are as follows: . 4. Select closed-loop pole (root as referred to in previous chapters) Io-] cations and develop the control law for the closed-loop system that} corresponds to satisfactory dynamic response (Sections 7.3 and 7.4) 2, Design an estimator (Section 7.5) Combine the control law and the estimator (Section 7.6) 4, Introduce the reference input (Sections 7.9.2 and 7.8). Figure 7.1 ‘control system design definition After working through the central design steps, we briefly explore the use of integral control in state space (Section 7.9). The final three sections of this chapter consider briefly some additional concepts pertaining to the state-space method; because they are relatively advanced, they may be considered optional to some courses or readers. A Perspective on State-Space Design In adcition to the transform techniques of root locus and frequency re- sponse, there is a third major method of designing feedback control sys- tems: the state-space method. In Chapter 2 we introduced the state- variable method of describing differential equations, In state-space de- sign, the control engineer designs a dynamic compensation by working directly with the state-variable description of the system. Like the trans- form techniques, the aim of the state-space method is to find a com- pensation D(s), such as that shown in Fig. 7.1, that satisfies the design specifications. Because the state-space method of describing the plant and computing the compensation is so different from the transform tech- niques, it may seem at first to be solving an entirely different problem. We selected the examples and analysis given toward the end of this chapter to help convince you that, indeed, state-space design results in a com- ensator with a transfer function D(s) that is equivalent to those D(s) ‘compensators obtained with the other two methods. Because it is particularly well suited to the use of computer tech- niques, state-space design is increasingly studied and used today by control engineers. Compensation Pia] roe) 4 A Dy bw oof 493, 494 Chapter7 rma fr Phase plane State-Space Design 7.1 Advantages of State Space ‘The idea of state space comes from the state-variable method of describing differential equations, which we introduced in Chapter 2. In this method the differential equations describing a dynamic system are organized as a set of first-order differential equations in the vector-valued state of the system, and the solution is visualized as a trajectory of this state vector in space. State-space ‘control design is the technique in which the control engineer designs a dynamic compensation by working directly with the state-variable description of the system, Thus far we have seen that the ordinary differential equations (ODEs) of physical dynamic systems can be manipulated into state-variable form. In the field of mathematics, where ODEs are studied, the state-variable form is, called the normal form for the equations, There are several good reasons for studying equations in this form, three of which are listed here: To study more general models: "The ODEs do not have to be linear or stationary. Thus by studying the equations themselves, we can develop ‘methods that are very general. Having them in state-variable form gives us ‘a compact, standard form for study. Furthermore. the techniques of state- space analysis and design easily extend to systems with multiple inputs and/or multiple outputs. Of course, in this text we study mainly linear time-invariant models with single input and output (for the reasons given earlier) © To introduce the ideas of geometry into differential equations: In physics the plane of position versus velocity of a particle or rigid body is called the phase plane, and the trajectory of the motion can be plotted as a curve in this plane. The state is a generalization of that idea to include more than two dimensions. While we cannot plot more than three dimensions, the concepts of distance and of orthogonal and parallel fines, as well as other concepts from geometry, can he useful in visualizing the solution of, ‘an ODE as a path in state space. # Toconnect internal and external descriptions: ‘The state of a dynamic sys- tem often directly describes the distribution of internal energy in the S/S- it is common to select the following as state variables: position (potential energy), velocity (kinetic energy), capacitor voltage {Clectric energy), and inductor current (magnetic energy). ‘The internal energy can always be computed from the state variables. By a system of analysis tobe described shortly, we ean relate the state to the system inputs fand outputs and thus connect the internal variables to the external inputs and to the sensed outputs. In contrast, the transfer function relates only the input to the output and does not show the internal behavior. The state form keeps the later information, which is sometimes important Use of the state-space approach has often been referred to as modern control design, and use of transfer-function-based methods such as root focus and fre- quency response has been referred to as elassieal control design. However, Section 7.2 Analysis of the State Equations 495, because the state-space method of description for ODEs has been in use for over 100 years and was introduced to control design in the late 1950s, it seems. somewhat misleading to refer to it as modern, We prefer to refer to the two approaches to design as state-space methods and transform methods Advantages of state-space design are especially apparent when the system to be controlled has more than one control input or more than one sensed output. However, in this book we will examine the ideas of state-space design using the simpler single-input-single-output systems. The design approach used for systems described in state form is “divide and conquer.” First we design the control asf all of the state were measured and available for use in the control law. This provides the possibility of assigning arbitrary dynamics for the system, Having a satisfactory control law based on full-state feedback, we introduce the concept of an observer and construct estimates of the state based on the sensed output. We then show that these estimates can be used in place of the actual state variables. Finally, we introduce the external reference-command inputs, and the structure is complete. Only at this point can we recognize that the resulting compensation has the same essential structure as that developed with transform methods Before we can begin the design using state descriptions, itis necessary to develop some analytical results and tools from matrix linear algebra for use throughout the chapter. We assume that you are familiar with elementary matrix concepts such as the identity matrix, triangula and the transpose of a matrix. We also assume you have some familiarity with the mechanics of matrix algebra, including adding, multiplying, and inverting matrices. More advanced results will be developed in Section 7.2 in the context of the dynamic response of a linear system. All of the linear algebra results used in this chapter are repeated in Appendix C for your reference and review. nnd cliagomal mi 7.2 Analysis of the State Equations In Chapter 2 we introduced and illustrated the process of selecting a state and. organizing the equations in state form. In this section we review that process and describe how to analyze the dynamic response using the state description, In Section 7.2.1 we begin by relating the state description to block diagrams and the Laplace transform description and to consider the fact that for a given system the choice of state is not unique, We show how to use this nonunique- ress to select among several canonical forms for the one that will help solve the particular problem at hand; a control canonical form makes feedback gains of the state easy to design. After studying the structure of state equations in Sec tion 7.2.2, we consider the dynamic response and show how transfer-function poles and zeros are related to the matrices ofthe state descriptions. To illustrate the results with hand calculations, we offer a simple example which represents the model of a thermal system. For more realistic examples, computer-aided control systems design software package such as MATL An is especially helpful; relevant Matias commands will be described from time to time. 496 Chapter 7 State-Space Design Figure 7.2 A block diagram representing Eq, (7.1) in conta form 7.2.1 Block Diagrams and Canonical Forms We begin with a thermal system that has a simple transfer function 5 Hs) s42 2 5 OF GQ) ~ THIs+ 44 t5H3 oy The roots of the numerator polynomial (s) are the zeros of the trans- fer function, and the roots of the denominator polynomial a(s) are the poles. Notice that we have represented the transfer function in two forms, as a ratio of polynomials and as the result of a partial-raction expansion. In order to develop a state description of this system (and this is a generally useful tech- nique), we construct a block diagram that corresponds to the transfer function (and the differential equations) using only isolated integrators as the dynamic elements. One such block diagram, structured in control canonical form, is drawn in Fig. 7.2. The central feature of this structure is that each state variable is connected by the feedback to the control input. Once we have drawn the block diagram in this form, we can identify the state description matrices simply by inspection; this is possible because when the output of an integrator is a state variable, the input of that integrator is the derivative of that variable, For example, in Fig. 7.2 the equation for the first state variable is fy = Im — 12 0 Continuing in this fashion, we get yen +20. These three equations can then be rewritten in the matrix form R= AK+ Bu 72) yx, 73) vote (s)—or vet OH Y I @Ha Mura F285 rl canonical orm Section 7.2 Analysis of the State Equations 497 [7 B} eff] 1) =[1 2, De=0 (4b) where and where the subscript ¢ refers to control canonical form. TWwo significant facts about this form are that the coefficients 1 and 2 of the numerator polynomial b(s) appear in the C, matrix, and (except for the leading term) the coefficients 7 and 12 of the denominator polynomial a(s) appear (with opposite signs) as the first row of the A, matrix. Armed with this, knowledge, we can thus write down by inspection the state matrices in control canonical form for any system whose transfer function is known as a ratio of numerator and denominator polynomials, If b(s) = bys"! + bas"? ++ By and a(s) = s"-4ays""" 4 ays"? +--+ ap, then the MATLAB Steps are: rum=b=[by be bol den=a=[1 a a an] [Ac Bo. Ge. Dz] = tf2ss(num, den) We read tf2ss as “transfer function to state space.” The result will be 1 0 0 4 Ag= B= (7.Say 0 0 0 Olt o Cal bo bh De (756) ‘The block diagram of Fig, 7.2 and the corresponding matrices of Eqs. (7.4) and (7.4b) are not the only way to represent the transfer function G(s). A. block diagram corresponding to the partial-fraction expansion of G(s) is given in Fig, 7.3. Using the same technique as before, with the state variables marked as shown on the figure, we can determine the matrices directly from the block diagram as being Anz + Baw ‘n+ Dat op melt] cre) 1), Du=0 (7.60) where 498 Chapter7 State-Space Design Figure 7:3, Block diagram for Eq. (7.1) in modal canonical form Modal form EXAMPLE 7.1 Y and the subscript m refers to modal canonical form. The name for this form de- rives from the fact that the poles of the system transfer function are sometimes called the normal modes of the system. The important fact about the matrices in this form is that the system poles (here —4 and —3) appear as the elements along the diagonal of the Aj matrix, and the residues, the numerator terms in the partial-fraction expansion (here 2 and ~1), appear in the Cx, matrix. "Expressing a system in modal canonical form can be complicated by two factors: (1) The elements of the matrices will be complex when the poles of the system are complex, and (2) the system matrix cannot be diagonal when the partial-fraction expansion has repeated poles. To solve the first problem we Express the complex poles of the partial-fraction expansion as conjugate paits in second-order terms so that all the elements remain real, The corresponding, ‘Avy matrix will then have 2 x 2 blocks along the main diagonal representing the local coupling between the variables of the complex-pole set. To handle the second difficulty we also couple the corresponding state variables, so that the poles appear along the diagonal with off-diagonal terms indicating the coupling ‘A simple example of this latter case is the satellite system from Example 2. whose transfer function is G(s) = L/s2. The system matrices for this transfer function in a modal form are rfid} oft} State Equations in Modal Canonical Form a) ‘A quarter ear mode!” [ given by c Eq, 2.12] with one resonant mode has a transfer function r+ L 1 Wea 7 AEDS 78) Ge)= = Sure Find state matrices in modal form describing this system, olution, ‘The transfer function has been given in real partia-fraction form. To get state-description matrices, we draw a corresponding block diagram with integrators Onl, asign the state, and write down the corresponding matrices. This process is not Figure 7.4 Block diagram for @ fourth-order system in ‘modal canonical form with shading indicating portion in control canonical form Stale descrintion and ouput Section 72. Analysis of the State Equations 499 (or SS) igs there are several spb sltion to the rblem ase but he Siero tl way A blosk Gagan wih satan signet a arate, iseveninPe nt ‘sce tat th second-order tex oepresent he complex pos has been ele incon canna form, ‘There area numberof ether palit an be wed avenue fori part. Ths parc fr alow ws to ote Joie en tates by apt: 00 0 0 1 To) Do 00 1 =0.(79) Thus far we have seen that we can obtain the state description from a trans- fer function in either control or modal form. Because these matrices represent the same dynamic system, we might ask: What is the relationship between the matrices in the two forms (and their corresponding state variables)? More generally, suppose we have a set of state equations that describe some physical system in no particular form and we are given a problem for which the control canonical form would be helpful. (We will see such a problem in Section 7.3.) Is it possible to calculate the desired canonical form without obtaining the trans- fer function first? To answer these questions requires a look at the topic of state transformations, Consider a system described by the state equations k=Fx+Gu, (7.10a) y= Bx4 Ju (7.106) As we have seen, this is not a unique description of the dynamic system. We consider a change of state from x (o a new state z that is linear transformation of x. Fora nonsingular matrix T we let xaT aay 500 Chapter 7 State-Space Design Transformation of state By substituting Eq. (7.11) into Eq, (7.10a), we have the equations of motion in terms of the new state 2: Th=FTs+ Gu (7.12) TIPT2+T ‘Gu. (7.12) = Ar+ Bu (7.126) where Ast RT (7.13a) B=T'G. (7.130) ‘Then we substitute Eq. (7.11) into Eq, (7.10b) to get the output in terms of the new state 2 y= HTz+Ju =Cz+ Du. where C=HT, D=s, ar) Given the general matrices F, G,and H and scalar J, we would like to find the transformation matrix T such that A. B, C, and D are in a particular form, for example, control canonical form. To find such a T we assume that A. B, C. and D are already in the required form, further assume that the transformation ‘T has a general form, and match terms. Here we will work aut the third-order cease; how to extend the analysis to the general case should be clear from the development. It goes ike this, First we rewrite Eq, (7.13a) as AT! = TR, If A isin control canonical form, and we describe T~! as a matrix with rows th, thand then [ ar Je}-[] (15) o 1 oo VERS" Lee Working out the third and second ows gives the matrix equations =F (7.16) (7.160) rolabity mati ‘tansformation to contra canonical frm | Section 7.2. Analysis of the State Equations 501 From Eq, (7.13b), assuming that B is also in control canonical form, we have the relation T'G=B 66) [:«]-[2] aun te!” Lo Combining Eqs (7.16) and (7.17), we get 4G =0, 1G SEG = 0, These equations can in turn be written in matrix form as tI6 FG FG]=[0 0 1] t=[0 0 1c"! (7.18) where the controllability matrix €=[G FG F°G]. Having ts,we can now go back to Eq. (7.16) and construct all the rows of T-! ‘To sum up, the recipe for converting a general state description of dimen- sion n to control canonical form is as follows: ‘¢ From F and G, form the controllability matrix © as e=(6 FG. Fig (7.19) ‘© Compute the last row of the inverse of the transformation matrix as =[0 0 ne" (7.20) ‘© Construct the entire transformation matrix as ure! ure? t ‘© Compute the new matrices from T-! using Eqs. (7.13) and (7.14). 502 Chapter 7 State-Space Design lable systems (nserver canonical orm When the controllability matrix © is nonsingular, the corresponding F and G matrices are said to be controllable, This isa technical property that usually holds for physical systems and will be important when we consider feedback of the state in Section 7.3. We will also consider a few physical illustrations of loss of controllability at that time. Because computing the transformation given by Eq. (7.21) is numerically difficult to do accurately, itis almost never done. The reason for developing this transformation in some detail is to show how such changes of state could be done in theory and to make the following important observation: One ean always transform a given state description to control canonical form if (and only if) the controllability matrix € is nonsingular If we need (0 test for controllability in a real case with numbers, we use a ‘numerically stable method that depends on converting the system matrices to “staircase” form rather than on trying to compute the controllability matrix. Problem 7.27 at the end of the chapter calls for consideration of this method. ‘An important question regarding controllability follows directly from our discussion so far: What is the effect of a state transformation on controllabil- ity? We can show the result by using Eqs. (7.19), (7.134), and (7.13b). The controllability matrix of the system (F, G) is Q=(G FG FG) (722) After the state transformation, the new description matrices are given by Eqs. (7.13a) and (7.13b), and the controllability matrix changes to G=[B AB. A''B) (7.23) “IT 'G] (7.236) 1g TPIT 'G rr ey. : (7.23) ‘Thus we see that @; is nonsingular if and only if Cy is nonsingular, yielding the following observation: [A change of state by # nonsingular linear transformation does ot change controllability We return once again to the transfer function of Eq. (7.1), this time to represent it with the block diagram having the structure known as observer canonical form (Fig. 7.5). The corresponding matrices for this form are [2] [2] =f Oo] (7.280) A, 24a) The significant fact about this canonical form is that all the feedback is from the output to the state variables. Figure 7.5 (Observer canonical form Section 7.2 Analysis of the State Equations 503 Let us now consider what happens to the controllability of this system as the zero at ~2 is varied, For this purpose we replace the second element 2 of B, with the variable zero location —z, and form the controllability matrix: C= [B. A.B] (7.25) (7.250) The determinant of this matrix is a function of det(@g) = 12-4 MT 2) ‘This polynomial is zero for <, = ~3 or ~4, implying that controllability is lost for these values, What does this mean? In terms of the parameter z,, the transfer function is OF0+9 If zy = ~3 or ~4, there is a pole-zero cancellation and the transfer function reduces from a second-order system to a first-order one. When :) = —3, for example, the mode at ~3is decoupled from the input and contra of this mode is lost Notice that we have taken the transfer function given by Eq. (7.1) and have given it two realizations, one in control canonical form and one in observer canonical form. The control form is always controllable for any value of the zero, while the observer form loses controllability if the zero cancels either of the poles. Thus, these two forms may represent the same transfer funetion, but it may not be possible to transform the state of one to the state of the other (in this case, from observer to control canonical form). While a transformation of state cannot affect controllability, the particular state selected from a transfer function can: Gis)= Controllability isa function of the state of the system and cannot be decided from a transfer function, 504 Chapter7 State “anstaoatio to rade fm Boenvectors Sgenvaues Mau og pace Design ‘To discuss controllability more at this point would take us too far afield. The closely related property of observability and the observer canonical form will be taken up in Section 7.5.1. A more detailed discussion of these properties of dynamic systems is given in the Appendix D, for those who would like to learn ‘We return now to the modal form for the equations, given by Eqs. (7.6a) and (7.6b) for the example transfer function, As mentioned before, itis not always possible to find a modal form for transfer functions that have repeated poles, so we assume our system has only distinct poles. Furthermore, we assume that the general state equations given by Eqs. (7.10a) and (7.10b) apply. We want to find a transformation matrix T defined by Eq, (7.11) such that the transformed Eqs (7.13) and (7.14) will be in modal form. In this ease, we assume that the A matrix is diagonal and that T is composed of the columns ty, t2, and ts, With this assumption, the state transformation Eq, (7.13a) becomes TA a 0 0 in aif’ e |- 0 0 ps Equation (7.26) is equivalent to the three vector-matrix equations b bh (7.26) 3 2 pe=Fy i In matrix algebra, Eq. (7.27) isa famous equation whose solution is known as the eigenvector/eigenvalue problem. Recall that t, is a vector, F is a matrix, and p; isa scalar. The vector t; is called an eigenvector of F, and p; is called the corresponding eigenvalue, Because we sav earlier that the modal form is, equivalent toa partial-fraction-expansion representation with the system poles, along the diagonal ofthe state matrix, it should be clear that these “eigenvalues” are precisely the poles of our system. ‘The transformation matrix that will convert the state description matrices to modal form has as its columns the eigenvectors of F as shown in Eq, (7.26) for the third-order case, Asit happens, there are robust, reliable computer algorithms to compute eigenvalues and the cigenvectors of quite large systems using the QR algorithm! In Maras the command p = eg(F) is the way to compute the poles ifthe system equations are in state form, Notice also that Eq. (7.27) is homogeneous in that, if t; is an eigenvector, so is at; for any scalar . In most cases the scale factor is selected so that the length (square root of the sum of squares of the magnitudes of the elements) is unity, Manan will perform this operation. Another option is to select the scale factors so that the input matrix B is composed of all I's The latter choice "This algorithm is part of MATLA@ and all other well-known computer-aided design packages Iris carefully documented in the software package LAPACK (Anderson ct al 1989). See also Strang (1988). Section 7.2 Analysis ofthe State Equations 50S is suggested by a partial-fraction expansion with each part realiz canonical form. If the system is real, then each element of F is real, and if p =a + jw isa pole, so is the conjugate, p* = 0 — jw For these eigenvalues the cigenvectors are also complex and conjugate. ‘It is possible to compose the transformation matrix using the real and complex parts of the eigenvectors separately so the modal form is real but has 2 «2 blocks for each pair of complex poles. Later we will see the result of the MatLas function that does this, but first let us look at the simple real-poles case. .d in control Transformation of Thermal System from Control to Modal EXAMPLE 7.2 Form Find the matrix to transform the contro} form matrices in Eq. (7.4) into the modal form of Eq, (7.6), Solution, According to Eqs. (7.26) and (7.27), we need first to find the eigenvectors savalues of the A, matrix. We take the eigenvectors to be The equations using the eigenvector on the left are (7 o][e]-e[2] ” Ty = Wer = pra (7.28) ” hsp (72%) Substituting Eg. (7.28) inte Bq. (7.28h) results in =Tptay ~ Wis) = pt tay + Toy + 2s =O, Pv Tp+12=0. pa 34. We have found (again!) that the eigenvalues (poles) are ~3 and —4; furthermore, Bq, (7.286) tells us that the Iwo eigenvectors are Co") [2] rt fos ssh ht temo BE) ray Tecan fo Ba 506 Chapter 7 State-Space Design f3] ay eet matrices of Eqs. (7.4) and (7.6) are related as follows: 1. Therefore, the AT, By = TB, 31) T, Dy =D, Ae Cy "These computations can be carried out using the MATLAB statements, T= [4-351 The next example has five state variables and in state-variable form is too complicated for handcalculations, However, itisa good example forillustrating the use of computer software designed for the purpose. The model we will use js based on a physical state after amplitude and time scaling have been done. EXAMPLE 7.3 Using Marian to Find Poles and Zeros of Tape-Drive s Find the eigenvalues ofthe system matrix forthe tape-drive control shown below. Also ‘compute the transformation of the equations of the tape drive in their given form to ‘modal canonical form, The system matrices are “ 0 o 0 0 0 0.1 035 01 OL 0.75 0 0 0 0 2 0 =|0 04 04 -04 -14 0 0 Oo ei id 00 0.0 1.0 60 0.0} Servomotor position output [05 0.0 0S 0.0 0.0] — Position at read/write head as output 02-02 02 0.2 00) tension output J =00. 2 To tng the inverse of a 2x 2 matrix. you need only interchange the elements subserpted “11” tind "22". change the sigs ofthe "12" and the "21" elements, and divide by the determinant 1 in Eg. (730) Section 7.2 Analysis of the State Equations 507 ‘The state vector is defined as 15 (tape position at capstan) 10 (Speed of the drive wheel) x =| 15 (position of the tape at the head) 23 (output speed) # (current into capstan motor) ‘The matrix Hy corresponds to making «5 (the position ofthe tape over the readiwrite head) the output, and the matrix Hy corresponds to making tension the output, Solution, ‘To compute the eigenvalues using MATLAB, we write Pp iat. which results in 0.6371 40.6691 0.6371 = 0.6669 6.0000 05075 0.9683 Notice that the system has al poles in the eft half-plane (LHP) except for one pole at the origin. This means that a step input will result in a ramp output, 0 we conclude the system has type I behavior “To transform to modal form, we use the MATLAB function canon: 9786 = S8(F,G, H9, J) [Am, Bm, Cm. Dm, TI] =canon(sysG, ‘modal’ The result of this caleulation is 0.6569 0.5371 9.0000 0.0000 0.0000 Am=An=| 0.0000 0.0000 -0.0000 0.0100 0.0000 | 1.0000 9.0000 0.0000 —0.5075 0.0000 0.0000 9.0000 0.0000 0.0000 0.9683 [0.8371 0.5669 G.0000«.0000 tn Notice that the complex poles appear in the 2 x 2 block in the upper left comer of Av and the real poles fall on the main diagonal of this matrix. The rest of the ealeulationS from canon are 3.0868, 12388 Bm =B. =| 4.0599]. om 9.9016 3.9341 [0.3268 0.2190 0.7071 -0.6625 0.6124); 1.1369) 11369 O74 3.0868; 1.1895 11895 3.2836 1.2388 Dy =0, T=T'=] 0.7376 5.4133 06767 1.3533 4.0599 09227 6.5022 0.9227 2.7962 9.9016 0.0014 0.1633 0.0014 0.0849 3.9341 Dm 508 Chapter? State. Nuruninw 1c Design It happens that canon was written fo compute the inverse ofthe transformation we are working with (as you can see from TI above). so we need to invert our MATLAB results. The inverse iscaleulated from T= inv) and results in [ 0.3626 0.1820 0.0000 0.2126 -0.3079 ‘The eigenvectors computed with [P}seig are <0.1748-£0.1420) 0.1748 0.14201 7071 -OA8TL 0.5887 0.0083 — 0.1035) 0.0083-+ 0.10357 0.0000 0.1236 —0.2850 V=| 08284402960) 0.8284 0.29601 0.7071 -0.8379 0.6360 0.3626 + 01820) 0.3626 — 0.18207 0.0000 0.2126 0.3079 10.0034 40.0022) 0.0034 — 0.00221 0.0000 —0.0075 0.2697 Notice thatthe first wo columns of the real transformation T are composed of the real and the imaginary parts ofthe first eigenvector inthe frst column of V. 11s this Step that causes the complex roots to appearin the 22 block inthe upper left ofthe Ay matrix. The veetorsin V are normalized to unit length, which resultsin non-normalized values in By and Cy. If we found it desirable to do so, we could readily find further transformations to make each clement of By, equal 1 or to interchange the order in whieh the poles appear. 7.2.2 Dynamic Response from the State Equations Having considered the structure of the state-variable equations, we now turn to finding the dynamic response from the state description and to the relationships between the state description and our earlier discussion in Chapter 6 of the frequency responseand polesand zeros, Let usbegin with the general equations of state given by Eqs. (7.10a) and (7.10b), and consider the problem in the frequency domain. Taking the Laplace transform of Fx (722) we obtain 5X(s) = x(0) = FX(s) + GUG). (7.33) which is now an algebraic equation. If we collect the terms involving X(s) on the left side of Eq, (7.33), keeping in mind that in matrix multiplication order Trastrtuction rom state EXAMPLE 7.4 Section 7.2. Analysis of the State Equations 509 is very important, we find that? (1 FIX(s) = GUG) + x0) 1 we premultiply both sides by the inverse of (sI— F), then X(s) = OI F)'GUG) + 1B) x), 34) The output of the system is Ys HX(s) + JUG), (7.354) = Hist ~ Fy 'GU(s} + His — Fy 'x(0) + JU%9). (7.35b) This equation expresses the output response to both an initial condition and an external forcing input. The coefficient of the external input is the transfer function of the system, which in this case is given by vo _ YO) 600) = PO MoI) 'G +, (7.36) Thermal System Transfer Function from the State Description Use Eg, (7.36) to find the transfer function ofthe thermal system deseribed by Eqs (74a) and (7:40) Solution, The state-variable description matrices of the system are [7 a} e=[] H=[I =O. To compute the transfer function according to Eq. (7.36), we form a-re['t7 ®] ae ‘and compute ‘The identity matrix I samateixot Ison the main diagonal and eros everywhere else: therefor nex 510 Chapter 7 State-Space Design We then substitute Eq, (7.37) into Eg. (7.36) to get wats -8)0) - (= WseD+R u au 1 — ul 138) weryen ey +2 1 crams 4) wa6FH ae The results can also be found using the MATLAB statements [num den] = ss2t(F.G.H.) and yield num = [0 1 2} and den 7 12), which agrees with hand caleulations, Because Eq. (7.36) expresses the transfer function in terms of the general state-space descriptor matrices F, G, H, and J, we are able to express poles and zeros in terms of these matrices. We saw earlier that by transforming the state matrices to diagonal form, the poles appear as the eigenvalues on the main diagonal of the F matrix. We now take a systems theory point of view to look at the poles and zeros as they are involved in the transient response of a system. ‘As we saw in Chapter 3, a pole of the transfer function G(s) isa value of generalized frequency s such that, if's = p,, then the system can respond to an initial condition as Kye", with no forcing function. In this context, p, is called ‘a natural frequency or natural mode of the system. If we take the state-space equations (7.10a) and (7.10b) and set the forcing function w (0 zero, we have xarx, (aD If we assume some (as yet unknown) initial condition x(0) = x 7.2) and that the entire state motion behaves at ‘quency, then the state can be written as x(¢ that -ording to the same natural fre- e?'xq, It follows from Eq. (7.41) K(1) = pyel"o = Fx = Fel", 7.43) Fx9 = pit. (744) We can rewrite Eq, (7.44) as (PA F)x% =0. (745) Taster function poles rom state equations Section 7.2. Analysis of th ate Equations S11 Equations (7.44) and (7.45) comprise the eigenvectorieigenvalue problem we saw in Eq. (7.27) with eigenvalues p, and in this case eigenvectors xp of the matrix F. If we are just interested in the eigenvalues, we can use the fact that for a nonzero xy, Eg. (7.45) has a solution if and only if det(p/l — F) = 0. (746) ‘These equations show again that the poles of the transfer function are the eigen- values of the system matrix F. The determinant equation (7.46) isa polynomial in the eigenvalues p; known as the characteristic equation, In Example 7.2 we computed the eigenvalues and eigenvectors of a particular matrix in control canonical form. As an alternative computation for the poles of that system, ‘we could solve the characteristic equation (7.46). For the system described by Eqs. (7.4a) and (7.4b), we can find the poles from Eq. (7.46) by solving det(st -) =0, (aay leer (rate) SO 4D412= (6496-44) 0. (te) This confirms again that the poles of the system are the eigenvalues of F. ‘We can also determine the zeros of a system from the state-variable de- scription matrices F, G, H, and J using a systems theory point of view. From this perspective aero isa value of generalized frequency s such that the system ‘can have a nonzero input and state and yet have an output of zero, If the input is exponential at the zero frequency z,, given by M(t) = oe’ (7.48) then the output is identically zero: ye) = 0. 749) The state-space description of Eqs. (7.48) and (7.49) would be ue = uye® " (7.50) Thus (751) or F | =o 732) and y SH Ju = Hea + Jie 512 Chapter 7 State-Space Design Transer function zr rom state eqatons EXAMPLE 7.5 Combining Eqs. (7.52) and (7.53), we get 1-F -G}fw]_[o ; ["a" F]E]-[0] a9 From Eq, (7.58) we can conclude that a zero of the state-space sytem isa value of <; where Eq. (7.54) has a nontrivial solution, With one input and one output, the matrix is square, and a solution to Eq, (7.54) is equivalent to a solution to cael a aci[ a Zeros for the Thermal System from a State Description Compute the zero(s) of the thermal system deseribed by Eq. (7.4). si mn. We use Fg, (7.55) 10 compute the zeros: +7 2 det} -1 Note that this result agrees with the 2er0 ofthe transfer function given by Eq. (7.1). The result can also be found using the MATLAB statements 8y8G = ss(Ac,Be,Ce,De} 7 = tzer0(sysG) and yields z= ~2.. ‘ Equation (7.46) for the characteristic equation and Eq. (7.55) for the zeros polynomial can be combined to express the transfer function in a compact form from state-description matrices as a(t ~~ det(st = FY Gs) (756) (Sce Appendix C for more details) While Eq. (7.56) is a compact formula for theoretical studies itis very sensitive to numerical errors. A numerically stable algorithm for computing the transfer function is described in Emami-Nacini and Van Dooren (1982b). Given the transfer function, we ean compute the fre queney response as Gi ja); and as discussed earlier, we can use Eqs (7.45) and (7-54) to find the poles and zeros, upon which the transient response depends, as we saw in Chapter 3 EXAMPLE 7.6 sua ss2tt Section7.2 Analysis of the State Equations $13 Analysis of the State Equations of a Tape Drive Compute the poles, zeros, and transfer function for the equations of the tape-drive servomechanism given in Example 7.3 Solution. The state equations are described by the system matrices: o 2 0 0 oO 0 01 035 01 OL 0.75 0 o 0 0 2 of, Gelo 04 04-04 140 0 0-003 000 0 1 = [0.0 00 L000 0.0) Servomotor position output [05 0.0 05 00 0.0] — Position at read/write head as output 02 02 0.2 02 00] — Tension output, There are two different ways to compute the answer to this problem. The most direct, is to use the MATLAB function ss2M (state space to transfer function), which will give the numerator and denominator polysomials disectl. Tis funetion permits multiple inputs and outputs the fith argument of the function tells which input i 1 be used. We have only one input here but must sil provide the argument, The computation of the transfer function from motor-current input to the servomotor position output is [IN2, 02] = ss2ti G, H2, J, 1) Which results in N2=[0 0.0000 0 0000 0.6000 1.2000) D2=[ 1.000 2.7500 3.2225 LABIS OAIN0 0.000 Similarly forthe postion atthe teadwrte head, the transfer function polynomials are computed by [N3, Da} = S824, 6, HB, J which results in N3=[0 0.0000 0.0000 6.7500 1.3500 1.2000) 3-= [1.0000 2.7500 3.2225 L.KSIS @.4180 0.0000) Finally, the transfer function to tension is INT, DM] = ss2tt G HT, J, 1) producing NT=[0 6.0000 0.1500 0.4300 0.3000 0.0000) DT = [1.0000 2.7500 3.2225 L.N8IS O.4180. 0.0000] 4 Chapter 7 Manu roots mua tzer0 State-Space Design Iti interesting to check to see whether the poles and zeros determined this way agree with those found by other means. For a polynomial we use the function roots: = 0.6371 + 0.6569: 0.6371 — 0.6669 rootsi(D3) = | 0.9683 0.5075 0.0000 Checking with Example 73, we confirm that they agn ‘Hove about the zeros?” We can find these by finding the roots of the numerator polynomial, We compute the roots of the polynomial NS: 24+ 4.1096 x 1071 2.4 = 4.1096 x 1077 0.9000 + 0.8888 =0,9000 — 0.8888 rootsiN3) Here we notice that roots are given with a magnitude of 10°, which seems inconsistent withthe values given for the polynomial. The problem is that MATLAB has used the very “mall leading terms ia the polynomial as real values and thereby introduced extraneous, roots that are forall practical purposes at infinity, The true zerosare found by truncating the polynomial to the significant values using the statement NOR = NBA: 6) toget NOR (0.7500 1.3500 1.200} sary [0.900 + 08888) rootinan=[ 990m +O ‘The other approach is to compute the poles and zeros separately and, if desired, combine these into. transfer function. The poles were computed with eigin Example 7.3 and are: 0.6371 + 0.66691 (0.6371 — 0.65691 0.0000 0.5075 0.9688 ‘The zeros can be computed by the equivalent of Eg. (7.54) with the function tzer0 (1vansmission zeros). The zeros depend on which output is being used, of course, and fre respectively given below. For the position of the tape at the servomotor as the ‘output, the statement: sysG2 = sstF,G,H2. J) ZER2 = trero(sysG2) yields ZER2 = ~2,0000. Estimatoobserver Section 7.3. Control-Law Design for Full-State Feedback 515 For the position of the tape over the read/write head as the output, the statement ‘9963 = ss(F. G.H3. J) ZERS = trero(sys63) 0.9000 + 0.8888" zee = (“haan oat | We note that these results agree with the valucs computed from the numerator polyao- rial NB above. And, finaly, forthe tension as output we use ‘s/8GT = ssiF,G,HT.J) ZERT = tzero(eysGT) toget 1.0000 zea =| -0.0000 2.0000, From these results we can write down, for example, the transfer funetion to position Kus a= BO 07802 +12 ~ SPREE S20 + Bs? SOARS 0754s +0.9:+ 0.8888) SSF OSD (6 + O96RNs + 0.637 0.67) (758) 7.3. Control-Law Design for Full-State Feedback One of the attractive features of the state-space design method is that it consists of a sequence of independent steps, as mentioned in the chapter overview. The first step, discussed in Section 7.3.1, is to determine the control. The purpose of the control law is to allow us to assign a set of pole locations for the closed-loop system that will correspond to satisfactory dynamie response in terms of rise lime and other measures of transient response. In Section 7.3.2 we will show how to introduce the reference input with full state feedback, and in Section 7.4 we will describe the process of finding the poles for good design, The second step—necessary if the full state is not available—is to design n normal form (sometimes called an observer), which computes an estimate of the entire state vector when provided with the measurements of the system indicated by Eq. (7.10b). We will examine estimator design in Section 7.5. 516 Chapter? State-Space Ds Figure 7.6 ‘Schematic diagram of state-space design ‘elements “The canto ia and ha estimator tgeter frm tne compensation Control tw U ‘Control law 7 eck] Esimator \ Matixof State vector The third step consists of combining the control law and the estimator. Figure 7.6 shows how the control law and the estimator fit together and how the combination takes the place of what we have been previously referring to as compensation. At this stage the control-law calculations are based on the estimated state rather than the actual state. In Section 7.6 we will show that this substitution is reasonable, and also that using the combined control law ‘and estimator results in closed-loop pole locations that are the same as those determined when designing the control and estimator separately. “The fourth and final step of state-space design is to introduce the reference input in such a way that the plant output will track external commands with acceptable rise-time, overshoot, and settling-time values, At this point in the design, all the closed-loop poles have been selected, and the designer is con- cerned with the zeros of the overall transfer function. Figure 7.6 shows the ‘command input r introduced in the same relative position as was done with the transform design methods; however, in Section 7.8 we will show how to introduce the reference at another location, resulting in different zeros and (usually) superior control 7.3.4 Finding the Control Law ‘The first step in the state-space design method, as mentioned earlier, i to find the control law as feedback of a linear combination of the state variables—that “1 w=-Kx=—-[Ki Ko Kall? (7.59) We assume for feedback purposes that all the elements of the state vector are at our disposal. In practice, of course, this would usually be a ridiculous assumption: moreover, a well-trained control designer knows that other design ‘methods do not require so many sensors. The assumption that al state variables re available merely allows us to proceed with this fist step. Figure 7.7 Assumed system for contro-aw design Cont characterise equation EXAMPLE 7.7 Section 7.3 Control-Law Design for Full-State Feedback 517 Equation (7.59) tells us thatthe system has a constant matrix in the state vector feedback path, as shown in Fig. 7.7. For an nth-order system there will be n feedback gains, Ki,.... Ky. and because there are n roots ofthe system, itis possible that there are enough degrees of freedom to select arbitrarily any desired root location by choosing the proper values of K). This freedom contrasts sharply with root-locus design, in which we have only one parameter and the closed-loop poles are restricted to the locus. Substituting the feedback law given by Eq, (7.59) into the system described by Eq. (7.10) yields &= FX - GK. (7.60) ‘The characteristic equation of this closed-loop system is, det{sI — oF ~ GK] =0. (701) ‘When evaluated, this yields an nth-order polynomial in » containing the gains Ky..-..Ky. The conirol-law design then consists of picking the gains K so. that the roots of Eq. (7.61) are in desirable locations. Selecting desirable root locations is an inexact science that may require some iteration by the designer. Issues in their selection are considered in Examples 7.7 to 7.9 as well as in Section 7.4. For now we assume that the desired locations are known, say, S126 2S Then the corresponding desired (control) characteristic equation is 8 = si )(s = 52)...(8 = 59) =O. (7.62) Hence the required elements of K are obtained by matching coefficients in Eqs. (7.61) and (7.62). ‘This forces the system’s characteristic equation to be identical (o the desired characteristic equation and the closed-loop poles to be placed at the desired locations, Control Law for a Pendulum Suppose you have a pendulum with frequ by ey «ay and a state-space description given ee ak both at ~2ay. In other words, you wish to double the natural frequeney and inerease the damping ratio ¢ from 0 to I S18 Chapter 7 State-Space Design Figure 7.8 Impuise response ofthe tundamped oseiiatar with full stato feedback (=) Solution, From Eq, (7.62) we find that (8) = (8-4 2a)? (74a) 2 des + dod (7.640) ee amn aff T(E EI Sa Kes taht = etfs = ¢ or (765) Equating the coefficients with like powers of s in Eqs. (7.64) and (7.65) yields the system of equations and therefore, More concisely, the control law is {305 dau} emt theintalcondiions m1 = 1.0, igre 7 shows the response ofthe xe oopsstem tt Sion = 104 ee ean oy 1 shown ey wll damped eaponses woud eget om ‘cing wo vous t= 2 The Marcas command pale was sed o general plot. Amel oa ya s 7 Tie vee Section 7.3, Control-Law Design for Full-State Feedback 519. Calculating the gains using the technique illustrated in Example 7.7 be- comes rather tedious when the order of the system is higher than 3. There are, however, special “canonical” forms of the state-variable equations where the algebra for finding the gains is especially simple. One such canonical form use ful in control law design isthe control eanonical form. Consider the third-order system ¥ tai + api + ayy = bili + bai + bow (7.6) Which corresponds to the transfer function Ys) bis? thay bbs _ bis) D0) ~ FFast tas tay Gs) Suppose we introduce an auxiliary variable (referred to as the partial state) €, which relates a(s) and b(s) as shown in Fig, 7.9(a), The transfer function from Uwoeis eye ) 76 UG) als) (7.68) Eta tank 1 as (7.99) Itis easy to draw a block diagram corresponding to Eq, (7.69) if we rearrange the equation as follows: F aif — ay ane +0 770) The summation indicates in Fig. 7.(b), where each & on the right-hand side is obtained by sequential integration of £. To form the output, we go back to Fig. 7.9(a) and note that Yes) = bIS)E(s), ay Which means ya bib + bok + bsé. an ‘We again pick off the outputs of the integrators, multiply them by {:}'s, and form the right-hand side of Eq, (7.66) by using a summer to yield the output as shown in Fig. 7.(c). In this case, all the feedback loops return to the point of the application of the input, or “control” variable, and hence the form is referred to as the contro! canonical form. Reduction of the structure by Mason's rule or by elementary block diagram operations verifies that this structure has the transfer function given by Gis) “1 This development is exactly the same for higher-order systems 520 Chapter? State-Space Design Figure 7.9 Derivation of control ‘canonical frm. sy For Oia eae emhe oe Taking the state asthe outputs ofthe three integrators nmbered, by con- vention, from the left “ wet (73) we obsain ur, (74) aon ial formin general, We may now write the matrices deseribing the control canon! form ing [Peet e-[tf os) ot 0 7 (7.756) Ackermann trmuta or pole Section 73 Control-Law Design for Full-State Feedback 521 ‘The special structure ofthis system matrixis referred to as the upper companion form because the characteristic equation is a(s) = s" +a;s"~!-+ays'"2---Lay, ‘and the coefficients of this monic “companion” polynomial are the elements in the first row of F.. If we now form the closed-loop system matrix F, ~ G.K, we find that : ox =| ro 0 (776) o By visually comparing Eqs. (7.75a) and (7.76) we see that the closed-loop char- acteristic equation is Car EK YS bas + KS Hy + Ky) =O. (7.77) Therefore, if the desired pole locations result in the characteristic equation given by acts) =" bans"! taps? os bay =O (7.78) then the necessary feedback gains can be found by equating the coefficients in Eqs. (7.77) and (7.78): Ky =a tay Miche = hoe morn | (AL) We now have the basis for a design procedure. Given a system of order nn described by an arbitrary (F, G) and given a desired nth-order monic char- acteristic polynomial a,(s), we (1) transform (F, G) to control canonical form (F.. G.) by changing the state x = Tz and (2) solve for the control gains by inspection using Eq. (7.79) to give the control law w = —K.z, Because this gain is for the state in the contol form, we must (3) transform the gain back to the original state to get K Analternative to this eee ‘method is given by Ackermann’s for- ‘Ackermann, 1972), which organizes the three-step process of converting mul to (F., G.), Solving for the gains, and converting back again into the following very compact form K=[0 =. 0 1]e-'a(F) (7.80) such that C=(G FG PG. PMG), (781) where @ is the controllability matrix we saw in Section 7.2, n gives the order of the system and the number of state variables, and @,(F) is a matrix defined as a) pank! pagPY2 +o tah. (782) where the a; are the coefficients of the desired characteristic polynomial Eq. (7.78). Note that Eq, (7.82) is a matrix equation 522 Chapter7 State-Space Design EXAMPLE 7.8 Ackermann’s Formula for Undamped Oscillator fa) Use Ackermann’s formula to solve for the gains for the undamped oscillator of Example 72. (b) Verify the caleulations with MATLAB for wy = 1 Solution, . (a) The desired characteristic equation is a. (s) = (s+ 20v) characteristic polynomial coefficients, iy, a = 48 (35) The controllability matrix is ex16 Fe)-[1 }] [to] Finally, we substitute Eqs. (7.84) and (7.83b) into Eq, (7.80) to get which yields Keim bol swf JES 33) K=[30j doo) Therefore which isthe same result we obtained previously. (b) The MATLAB statements F = [0 1;-wo-wo 0}; G=(0:1) pe = [2+w0;-2-W0}: K = ackenG.p0) yield K=[3- 4], which agrees with hand calculations. Therefore, the desired (783a) (7.830) ss) sue acker, place An erample of weak onlay Section 7.3 Control-Law Design for Full-State Feedback 523 ‘As was mentioned earlier, computation of the controllability matrix has very oor numerical accuracy, and this carries over to Ackermann’s formula. Equa- tion (7.80), implemented in Matas with the function acker, can be used for the design of single-input-single-output systems with a small (= 10) number of state variables. For more complex cases a more reliable formula is avail- able, implemented in MaTLas with place. A modest limitation on place is that, because itis based on assigning closed-loop eigenvectors, none of the de- sired closed-loop poles may be repeated: that is, the poles must be distinct,’ a requirement that does not apply to acker. ‘The fact that we can shift the poles of a system by state feedback to any desired location is a rather remarkable result. The development in this section reveals that this shift is possible if we ean transform (F, G) to the control form (F,, G.), which in turn is possible if the system is controllable, In rare instances the system may be uncontrollable, in which case no possible control will yield arbitrary pole locations. Uncontrollable systems have certain modes, or subsystems, that are unaffected by the control. This usually means that paris of the system are physically disconnected from the input. For example, in modal canonical form for a system with distinct poles, one of the modal state variablesisnot connected to the input ifthereisazero entryin the By matrix. A ‘good physical understanding of the system being controlled would prevent any attempt to design a controller for an uncontrollable system, As we saw earlier, there are algebraic tests for controllability; however, no mathematical test can replace the control engineer's understanding of the physical system, Often the physical situation is such that every mode is controllable to some degree, and, while the mathematical tests indicate the system is controllable, certain modes are so weakly controllable that designs to control them are virtually useless. Airplane controlisa good example of weak controllability of certain modes. Pitch plane motion x, is primarily affected by the elevator 8, and weakly af- fected by rolling motion x,.. Rolling motion is essentially only affected by the ailerons 6,. The state-space description of these relationships is [e}-[5 JEM elk} es ‘where the matrix of small numbers ¢ represents the weak coupling from rolling motion to pitching motion, A mathematical test of controllability for this sys- tem would conclude that pitch plane motion (and therefore altitude) is eontrol- lable by the ailerons as well a by the elevator! However, itis impractical to attempt to control an airplane's altitude by rolling the aircraft with the ailerons. Another example will illustrate some of the properties of pole placement by state feedback and the effects of loss of controllability on the process. * One may get around this restriction by moving the repeated poles by very small amounts 10 make them distinc. 524 Chapter 7 EXAMPLE 7.9 State-Space Design How Zero Location Can Affect the Control Law A specitie thermal system is deseribed by Eq, (7.24) in observer canonical form with a (a) Find the state feedback gains necessary for placing the poles ofthis system at the roots of «2 + 26048 +02—that is, at —Gay, + jay T 6 (b) Repeat the computation with MATLAB using the parameter values zy = 2 and ay = 2 radisec. Solution. (a) The state description matrices First we substitute these matrices into Eq (761) to get the closed-loop characteristic jon in terms of the unknown gains and the zero position: PATH K, — 20Kz)s +12 ~ Kal Next we equate this equation to the desired characteristic equation to get the equations equ +12) Kiso The solutions to these equations are )4 1220 ~ 7) ra (14604 — 37 Got G44) (b) The MarLan statements “ 20}: Pe = rootel{t 24); K = place(Ao.Bo,pe) can be used to find the solution 3.80 0.60}, which agrees with hand calculations. IFthe zero were close to one of the open-loop poles, say =» = ~2.99 then we find K = [2052.5 ~ 688.1), Two important observations should be made from this example. The firsts that the gains grow as the zero zy approaches either —3 or 4, the values where this system loses controllability. In other words, as controllability is almost lost the control gains become very large. ‘Te system has to work harder and harder to achieve control as controllability slips away, nin caeation for eference Section 7.3 Control-Law Design for Full-State Feedback 525 ‘The second important observation illustrated by the example is that both Ky and Kz grow as the desired closed-loop bandwidth given by «, is increased, From this, we conclude that To move the poles a long way requires large gains ‘These observations lead us to a discussion of how we might go about selecting desired pole locations in general. Before we begin that topic we will complete the design with full-state feedback by showing how the reference input might be applied to such a system and what the resulting response characteristics are. 7.3.2 Introducing the Reference Input with Full-State Feedback ‘Thus far, the control has been given by Eq. (7.59), or w= Kx. In order to study the transient response of the pole-placement designs to input commands, it is necessary to introduce the reference input into the system. An obvious way to do this is to change the control to u = —Kx +r, However, the system will now almost surely have a nonzero steady-state error to a step input, The ‘way to correct this problem is to compute the steady-state values of the state and the control input that will result in zero output error and then force them to take these values, If the desired final values of the state and the control input are x,, and w,,, then the new control formula should be Wty Kix x), (7.86) so that when x = Xj, (no error), u = 1,. To pick the correct final values, we ‘must solve the equations so that the system will have zero steady-state error 10 any constant input. The system differential equations are the standard ones: k= Prt Gu q. ) yal Ju (787) In the constant steady-state, Eqs (7.87a) and (7.876) reduce to the pair O= Fx, + Gn, (7.880) du Hay + Jit (7880) We want to solve for the values for which 9x, =r, for any value ofr... To do this we make X,5 =Neryy and tty, = Nyra.- With these substitutions we can write Eq, (7.88) as a matrix equation; the common factor of r,, cancels out 10 ive the equation for the gains: [m S][s) Et] ea 526 Chapter? State-Space Design Contra equation with rofeonce input Figure 7.10 Block diagram for Introducing the reference input with ful-state feedback: (a) with state ‘and control gains; {b) wath 2 Single composite gain EXAMPLE 7.10 ‘This equation can be solved for Ny and Ni, to get feJ-G oT) 0) With these values we finally have the basis for introducing the reference input so as Lo get zero steady-state error to a step input: = Nur — Kix —Nyr) (7.900) = Kx 4 (Ny + KNpr (7.90) The coefficient of r in parentheses is a constant that can be computed before- hhand. We give it the symbol 4’, so w= Kx + Wr 1) The block diagram of the system is shown in Fig. 7.10 tHe TL, Get. Fe] | er Introducing the Reference Input Compute the necessary gains for zero steady-state error to a step command at x, and plot the resulting unit step response forthe oscillator in Example 7.7 with aay = 1 to} Solution, We substitute the matrices of Eg. (7.63) (with ay = Land I scause y = xi) into Eg, (7.89) to get 0 © [* 0 Ie] [*] (792) i 100 1 ‘The solution is (x = a\ in MATLAB, where @ and b are the left and right-hand sides matrices respectively), Figure 7.11 Step response of oscilator toa reference input EXAMPLE 7.11 De moor Ea 262 Section7.3 Control-Law Design for Full-State Feedback 527 eee seer Time (ee) nd, for the given control law; K = (30 4a] =[3 4]. A Net KN, =4 (793) The corresponding step response (using the MATLAB step command) is plotted in Fig 7.11 Note that there are two equations for the control: Eqs, (7.90) and (7.91). Whil these expressions are equivalent in theory, they differ in practical implementa- tion in that Eq. (7.90) isusually more robust to parameter errors than Eq. (7.91), particularly when the plant includes a pole at the origin and type 1 behavior is possible. The difference is most clearly illustrated by the following example, Reference Input to a Type 1 S sstem: DC Motor Compute the input gains necessary to introduce a reference input with zero steady state error toa step for the DC motor of Example 5.1, which in state-varible form is described by the matrices: Pei 0 i a H=[1 0} J edback gain is [Ky Ko} Assume that the state Solution, If we substitute the system matrices of this example into the equation for the input gains, Eq, (7.89), we find that the solution is 528 Chapter 7 State-Space D Figure 7.12 Alternative structures for intzoducing the reference input. (a) a, (7.90) (©) Ea. (7.91) nos) With these values the expres to sion for the control using Ny and N, [Eq (7.90)] reduces RG 7) Boxe while the one using (Eq, (7.91)] becomes Ky) = Kosa + Kir The block diagrams for the systems using each of the control equotions are given in Fig. 7.12. When using Eq, (7:1), as shown in Fig. 7.12(b), itis necessary to multiply the input by a gain Ki(= ¥) exactly equal to that used inthe feedback sf these two gains do not match exactly. thee wil bea steady-state error. On the other hand if we use Eq. (7.0) as shown in Fi, 7-12(a), there is only one gain to be used on the difference between the reference input and the first state, and zero steady-state error will result even i this gain i slightly in error. The system of Fig 7.12(a) is more robust than the system of Fig, 7.1200) With the reference input in place, the closed-loop system has input r and output y. From the state description we know that the system poles are at the eigenvalues of the closed-loop system matrix, F GK. In order tocompute the closed-loop transient response, itis necessary to know where the closed-loop zeros of the transfer function from r to y are. They are to be found by applying Eq, (7.55) t0 the closed-loop description, which we assume has no direct path from input w to output y so that J =0. The zeros are values of s such that aft K) NG af Re (7.98) methods of poe seein Section 7.4. Selection of Pole Locations for Good Design 529 We can use two elementary facts about determinants to simplify Eq. (7.94). In the first place, if we divide the last column by N, which is a scalar, then the point where the determinant is zero remains unchanged. The determinant is also not changed if we multiply the last column by K and add it to the first (block) column, with the result that the GK term is cancelled out, Thus the matrix equation for the zeros reduces to [at 8] Equation (7.95) is the same as Eq, (7.55) for the zeros of the plant before the feedback was applied. The important conclusion is as follows: (7.95) When fullstate feedback is used as in Eq, (7.900) oF (7.91), the zeros remain unchanged by the feedback, 7.4 Selection of Pole Locations for Good Design ‘The first step in the pole-placement design approach is to decide on the closed- loop pole locations. When selecting pole locations, itis always useful to keep in mind that the control effort required is related to how far the open-loop poles are moved by the feedback. Furthermore, when a zero isnear a pole, thesystem may be nearly uncontrollable and as we saw in Section 7.3, moving such poles requires large control gains and thus large control effort; however, the designer is able to temper the choices to take control effort into account. Therefore, a pole-placement philosophy that aims to fix only the undesirable aspects of the open-loop response and avoids either large increases in bandwidth or efforts to move poles that are near zeros will typically allow smaller gains and thus smaller control actuators than a philosophy that arbitrarily picks all the poles without regard to the original open-loop pole and zero locations, Inthis section we discuss two techniques to aid inthe pole-selection process. ‘The first approach—dominant second-order poles—deals with pole selection without explicit regard for their effect on control effort; however, the designer is able to temper the choices to take control effort into account, ‘The sec- ‘ond method (called optimal conirol, or symmetric root locus) does specifically address the issue of achieving a balance between good system response and control effort 7.41 Dominant Second-Order Poles The step response corresponding to the second-order transfer function with, complex poles at radius «, and damping ratio ¢ was discussed in Chapter 3. The rise time, overshoot, and settling time can be deduced directly from the 530. Chapter7 State-Space Design EXAMPLE 7.12 suas acker pole locations. We can choose the closed-loop poles for a higher-order system fas a desired pair of dominant second-order poles, and select the rest of the poles to have real parts corresponding to sufficiently damped modes, so that the system will mimic a second-order response with reasonable control effort. ‘We also must make sure that the zeros are far enough into the left half-plane (LHP) to avoid having any appreciable effect on the second-order behavior. A system with several lightly damped high-frequency vibration modes plus two rigid-body low-frequency modes lends itself to this philosophy. Here we can pick the low-frequency modes to achieve desired values of a, and ¢ and select the rest of the poles to increase the damping of the high-frequency modes, while holding their frequency constant in order to minimize control effort. To illustrate this design method we obviously need a system of higher than second order; we will use the tape drive servomotor described in Example 7.3. Pole Placement as a Dominant Second-Order $ yysrem Design the tape servomotor by the dominant second-order poles method to have no more than 5% overshoot and arise time of no more than 4 sec. Keep the peak tension as low as possible. Solution, From the plots of the second-order transients in Fig. 3.23, a damping ratio = 0.7 will meet the overshoot requiseaent sd, ox this damping ratio, arise time of 4 sec suggests a natural frequency of about 1/15, There are five poles in all, so the other three need to be placed far to the left of the dominant pair. For our purposes, “far ‘means the transients due to the fast poles should be over well before the transients due to the dominant poles, and we assume a factor of 4 in the respective undamped natural frequencies to be adequate. From these considerations, the desired poles are given by po = [-0.707 +0.707 #|;-0.707 0.707 ej; 4:4: —a/1.5; (796) With these desired poles, we ean use the function acker with F and G from Example 73, Eq, (759), to find the control gains Ky = [85123 203457 1.4911 ~7.8821 6.1927] «rn ‘These are found using the MATLAB statements (02000;-.1 95.1 .1.75;00020;4.4-4 -1.40;0-.0800 ~1} (:0:0:0;15, pe = [.707+.707- K2 = ackerGpe) The step response and the corresponding tension plots for this and another design (40 be discussed in Section 7.4.2) ate given in Figs. 7.13 and Figs, 7.14. Notice that the rise time is approximately 4 see und the overshoot is about 5%, as specified. Because the design process is iterative, the poles we selected should be seen as only a first step, to be followed by further modifications to meet the specifications as accurately as necessary. For this example we happened to select adequate pole locations on the first try. Figure 7.13 Step responses ofthe tape servomotor designs Figure 7.14 Tension pots for tape servomotor etep reeponeoe design Section 7.4 Selection of Pole Locations for Good Design 531 Lo Dominant secondedee 02 ° - Oa cet Dormia second-order Time see) 7.4.2 Symmetric Root Locus (SRL) A most effective and widely used technique of linear control systems design is the optimal linear quadratic regulator (LQR). The simplified version of the LOR problem isto find the control sueh that the performance index a= [ete +A (798) Symmetic oot oeus SAL equation 32 Chapter 7 State-Space Design is minimized for the system X= Pe} Gu, (7.99) Hx, (7.99) where p in Eq. (7.98) is a weighting factor of the designer's choice. A re. markable fact is that the control law that minimizes J is given by linear-state feedback w= Kx, (7.00 Here the optimal value of K is that which places the closed-loop poles at the stable roots (those in the LHP) of the symmetric root-locus (SRL) equation (Kailath, 1980) 1+ pGol—NGots) 0, ony Where Go is the open-loop transfer function from w to z sce) - Zi) © Gols) = 2) = nyt - Fy tg = XO G9) = Tey = NGI BG = Dey (7.102) Note that this isa root-locus problem as discussed in Chapter 5 with respect (o the parameter p, which weights the the relative cost of (tracking error) ? with respect to the control effort w- in the performance index Eq, (7.98). Note also that and —s affect Eq, (7.101) in an identical manner; therefore, for any root 5 of Eq. (7.101), there will also be a root at —s). We call the resulting. root locus a symmetric root locus (SRL). since the locus in the LHP will have @ mirror image in the right-half-plane (RHP): that is, there is symmetry with respect to the imaginary axis. We may thus choose the optimal closed-loop poles by first selecting the matrix Hy. which defines the tracking error and which the designer wishes to keep small, and then choosing, which balances the importance of this tracking error against the control effort, Notice that the output we select as tracking error does or need to be the plant Sensor output. ‘That is why we call the output in Eq, (7.99) z rather than y Selecting a set of stable poles from the solution of Eq. (7.101) results in de sired closed-loop poles, which we can then use in a pole-placement calculation such as Ackermann’s formula (Eq. (7.80)] to obtain K. As with all root loci for real transfer functions Go, the locus is also symmettic with respect to the real axis; thus there is symmetry with respect to both the real and imaginary axes. ‘We can write the SRL equation in the standard root-locus form NON) ? DED (7.108) obtain the locus poles and zeros by reflecting the open-loop poles and zeros of the transfer function from U 10 Z across the imaginary axis (which doubles the number of poles and zeros), and then sketch the locus. Note that the locus could be either a 0° or 180. locus, depending on the sign of Go(—s)Go(s) in EXAMPLE 7.13 Figure 7.15 ‘Symmetric rot lacus for first-order system EXAMPLE 7.14 Section 7.4 Selection of Pole Locations for Good Design 533 Eq. (7.101), A quick way to determine which type of locus to use (0° or 180° is to pick the one that has no part on the imaginary axis. The real-axis rule of root locus plotting will reveal this right away. For the controllability assumptions we hhave made here, plus the assumption that all the system modes are present in the chosen output z, the optimal closed-loop system is guaranteed to be stable; thus no part of the locus can be on the imaginary axis. The following examples illustrate the use of the SRL, Symmetric Root Locus for Servo Speed Control Plot the SRL for the following servo speed control system with ¢ b= ~ay bu (7.104) 1 Gauls) = 7.1040) Gu) = (7.1040) Solution. The SRL equation (Eq, (7.101)] for this example is L +o (7.405) Gaeta The SRL, shown in Fig. 7.15, isa 0° locus, The optimal (sable) pole can be determined explicitly inthis ease as Vite. (7.108) Thus the closed-loop root location that minimizes the performance index of Eq, (7.98) lies on the real axis atthe distance given by Eq, (7.106) and is always to the left of the ‘open-loop root. pro pao peo po SRL Design for Satellite Attitude Control Draw the SRL for the satellite system with Solution, The equations of motion are Edel) v0 yell Ox (rs) 534 Chapter 7 State-Space Design Figure 7.16 Im) Symmetric rot locus for the satelite ‘We then caleulate trom Eqs, (7,107) and (7.108) so that 1 Guts) crn the symmetsie 180" loot are shown in Fig. 7.16, The MATLAB statements 10 generate the SRL are numG@ = [1 denGG = convift 00}{1 0 0): sySGG = tnumGG,denGG); rlocus(sysGG) It is interesting to note that the (stable) closed-loop poles have damping of ¢ = 0.707, We would choose twostable roots for a given value of p—for example for p = 4.07—on the SRL and use them for pole-placement and control-law design Choosing different values of p can provide us with pole locations that achieve varying balances between a fast response (small values of ':2dt and a low control effort (small values of fw? d). Figure 7.17 shows the design- tradeoff curve forthe satellite (double-integrator) plant [Eq. (7.7) for various values of p ranging from .01 to 100, ‘The curve has two asymptotes (dashed lines) corresponding to low (large p) and high (small p) penalty on the control usage. In practice, usually a value of p corresponding to a point close to the knee of the tradeoff curve is chosen, This is because it provides a reasonable compromise between the use of control and the speed of response. For the satellite plant, the value of p = 1 corresponds to the knee of the curve. In this case the closed-loop poles have a damping ratio of ¢ = 0.707! Figure 7.18, shows the associated Nyquist plot, which has a phase mare 65° and infinite gain margin, These excellent stability properties are a general feature of LOR designs. Figure 7.17 Design trade-off curve for satelite plant Figure 7.18 NNyguist plot for LOR design EXAMPLE 7.15, Section 7.4 ‘lection of Pole Locations for Good Design 535 25-20-15 10 05 0 OS 10 Real axis It is also possible to locate optimal pole locations for the design of an open-loop unstable system using the SRL and LOR method. SRL Design for an Inverted Pendulum Draw the SRL for the linearized equations of the simple inverted pendulum with « = 1 Take the output, z, to be the sum of twice the position plus the velocity (so as to weight or penalize both position and velocity) Solution, The equations of motion are [3 de [5] or 536. Chapter 7 Figure 7.19 Symmetric root locus for the inverted pendulum State-Space Design For the specified output of 2x position plus Velocity we let the tracking error be 1 cany We then caleulate from Eqs. (7.110) and (7.111) so that Gals) =~ (7.2 ‘The symmetric 0° loci are shown in Fig. 7.19. The MATLAB statements to genes SRL are (for w, = 1). numGG = convé{t 2}-{-1 2 denGG = convift 0 = 1){1 0 ~ 1); sy8GG = tnum@G dena), Hocus(sysGG); For p= 1, we find that the closed-loop poles are at —1,36: /0,606, corresponding toK =[-223 ~2.73]. If we substitute the system matrices ofthis example into the equation for the input gains, Eq, (7.89), we find that the solution is ‘With these values the expression for the control using Ny and, (Eq, (7:90)] the controller reduces to w= -Kx The corresponding step response for postion is shown in Fig. 7.20. Figure 7.20 Step response for the inverted pendulum Matar Bryson’ ule Section 7.4 Selection of Pole Locations for Good Design 537 0 os 1 aS 2 as eas ‘Asa final example in this section, we consider again the tape servomotor and introduce LOR design using the computer directly to solve for the optimal control law. From Eqs. (7.98) and (7.100) we know the information needed to find the optimal control is given by the system matrices F and G and the output matrix Hj, Most computer-aided software packages, including MATLAB, use a more general form of Eq, (7.98): = [ox +urRwnat (7.113) Equation (7.113) reduces to the simpler form of Eq. (7.98) if we take Q = HTH, and R = 1. The direct solution for the optimal control gain is the Maran statement K =IqrF,G.0.R) aaiay One reasonable method to start the LOR design iteration is suggested by Bryson’s rule (Bryson and Ho, 1969). In practice, an appropriate choice 10 obtain acceptable values of x and u is to initially choose diagonal matrices Q and R such that Qi = L/maximum acceptable value of [x2] /maximum acceptable value of [ ‘The weighting matrices are then modified during subsequent iterations to achieve an acceptable tradeoff between performance and control effort. 538. Chapter 7 State-Space Design EXAMPLE 7.16 LOR Design for a Tape Drive (a) Find the optimal control forthe tape drive of Example 7.6, wsing the positon xs as ) the output for the performance index. Let p = 1. Compare the results with that of dominant second-order obtained before. (b) Compare the LOR designs for p = 0.1, 1,10 Sok (a) All we need to do here is to substitute the matrices into Eq, (7.114), form the feedback system, and plot the response. The performance index matrix is the scalar & = 1: the most dificult part of the problem is finding the state-cost matrix @. With the outputcost variable z =.x5, the output matrix from Example 73 is Hy=[05 0 05 0 6] and with p = 1, then the required matrix is Q=HIH, 025 0 025 0 0° dD 0 0 90 025 0 025 0 0 0 0 0 00 “The gain is given by MATLAB, using the statements 95.1.1 .7500020:4.4-4 ~1.450-0300 —1 thosH3"H3: aiF.a.R) K = [0.6526 2.1667 0.3474 0.5976 1.0616] (ris) The results ofa postion step and the corresponding tension are plotted in Figs. 7.13 sand 7.14 (using step) with the earlier responses for comparison. Obviously, there is a vast range of choice forthe elements of Q and R. so substantial experience is needed in order to use the LOR method effectively (b) The LOR designs may be repeated as above with the same Q and R, but with 1p 011, 10. Figure 721 shows a comparison of position step and the corresponding fension for the three designs. As seen from the resulls the smaller values of ‘correspond to higher cost on the control and slower response, whereas the larger ‘Values of » correspond! to lower cost on the control and relatively fast response. Figure 7.24 (@) Step response of the ‘ape servomotor for LOR designs, (o) Corresponding tension for tape servomotor step responses Section 7.4 Selection of Pole Locations for Good Design 539 p= pet Time mee) @ 010 0s 010 “Tape tension. T oas Time mee) Limiting Behavior of LQR Regulator Poles It is interesting to consider the limiting behavior of the optimal closed-loop poles as a function of the root-locus parameter (i.e., p), although, in practice, neither case would be used. “Expensive control” case (p -> 0). Equation (7.98) primarily penalizes the use of control energy. If the control is expensive, then the optimal control does not move any of the open-loop poles except for those which are in the right half-plane (RHP). The poles in the RHP are simply moved to their mirror images in the LHP. The optimal control does this to stabilize the system using 540 Chapter 7 State-Space Design n and phase margins minimum control effort and makes no aitempt to move any of the poles of the system in the LHP. The closed-loop pole locations are simply the starting points on the SRL in the LHP. The optimal control does not speed up the response Of the system in this case. For the satellite plant, the vertical dashed line in Fig. 7.17 corresponds to the “expensive control” case and illustrates that the very low control usage results in a very large error in © “Cheap control” case (p + oe). In this case control energy is no object and arbitrary control effort may be used by the optimal control law. The con- trol law then moves some of the closed-loop pole locations right on top of the zeros in the LHP. The rest are moved to infinity along the SRL asymptotes. If the system is nonminimum phase, some of the closed-loop poles are moved to mirror images of these zeros in the LHP as shown in Example 7.15. The rest Of the poles go to infinity and do so along a Butterworth filter pole pattern as shown in Example 7.14. The optimal control law provides the fastest possible response time consistent with the LOR cost function. ‘The feedback gain ma- trix K becomes unbounded in this case. For the double-integrator plant, the horizontal dashed line in Fig. 7.17 corresponds to the “cheap control” case Robustness Properties of LOR Regulators It has been proved (Anderson and Moore, 1990) that the Nyquist plot for LOR design avoids a circle of radius one centered at the —1 point as shown in Fig. 718. This leads to extraordinary phase and gain margin properties. It can be shown (see Problem 7.30) that the return difference must satisfy [+ K(jol FY 'G| > 1 (7.116) Let us rewrite the loop gain as sum of its real and imaginary parts: LG@) = Kol ~ F" Re(L{jo)) + jlm(Lt jo) ay Equation (7.116) implies that (Re(L(o)] +0? + [Im(LGioy 21 (7.118) which means that the Nyquist plot must indeed avoid a circle centered at - with unit radius. This implies that } 0 and Ry > 0. The estimator design yields = FR + Gu + Liy — 5). (7.176) bom, (7.1766) 570 Chapter 7 State-Space Design Prat iersion LER fr nanmininum-prase sisters EXAMPLE 7.26 resulting in the usual dynamic compensator Das Kit F+6K + La) 'L a7) We will now treat the noise parameters Ry, and Ry, as design “knobs” in the dynamic compensator design. Without loss of generality let us choose R, = IT? and R, = 1. For loop transfer recovery, assume F = gG where q isa scalar design parameter. The estimator design is then based on the specific design parameters Ry, and R,. It can be shown that for a minimum-phase system as q becomes large (Doyle and Stein, 1979) lim, D.(s)G(s) = Kis — FG (7.178) and the convergence is pointwise in s and the degree of recovery can be arbitrar- ily good. This design procedure in effect “inverts” the plant transfer function in the limit as. q — oc, lim De(s) = K(sI = FY'GG™"(0), (7.179) and this is precisely the reason that full loop transfer recovery isnot possible for a nonminimum-phase system. This limiting behavior may be explained using. the symmetric root loci. As q —> 00, some of the estimator poles approach the zeros of Gas) = Hot Fy 'T, (7.180) and the rest tend to infinity® [see Eqs. (7.155) and (7.156)}. In practice, the LTR design procedure can still be applied to a nonminimum-phase plant. The degrce of recovery will depend on the specific locations of the nonminimum- phase zeros. Sufficient recovery should be possible at many frequencies if the RHP zeros are located outside the specified closed-loop bandwidth. Limits on achievable performance of feedback systems due to RHP zeros are discussed in (Freudenberg and Looze, 1985). We will next illustrate the LTR procedure by a simple example. LTR Design for Satellite Auitude Control Consider the satellite system with state-space deseription OL fo i 0 a) 6-[7] H=(1 0), J=0 na Butterworth configuration, Figure 7.43 Frequency response plats for UTR design Section 7.7 Loop Transfer Recovery (LTR) 571 (a) Design an LOR controller with Q = pHH ond & = 1, p = 1,and determine the Joop gain. (b) Then design a compensator which recovers the LOR loop gain of part (a) using the loop transfer recovery (LTR) technique for q = 1, 10.100 (©) Compare the different candidate desians in part (b) with respect to the actuator activity due to additive white gaussian sensor noise Solu Using lar the selected LOR weights result in the feedback gain K [I 1.414}, The loop transfer function is, 114s +0707) Kol -Fy'G = AA magnitude frequency response pt of this LOR loop gai is shown in Fig. 743. For theestimator design wsingige.let F = gG,R, = FT, and R, = 1 and choose q = 10, resulting inthe estimator gain ae -["to'] ‘The compensator transfer function is Ds) = Kisl — F +GK + LH) 'L 155.565 + 0.6928) 155 564s + 06828) BH 15.5865 +12 ~ GFT TING + TIT TTT and the loop transfer function is 155.565 40.6428) PANGS) = FETT + FLT + TIT TTD. 5 au, ote a= 10 3 10 q=10 a=t wt 1 10) =120| Phase (et) | =270) ry stead) 572. Chapter 7 Mone gr Ne ige Maruebode Manus margin Ms value State-Space Design igure 743 shows the frequency response of the loop transfer function for several values, ‘fy (q = 1, 10.100) along with the ideal LOR loop transfer function frequency response, ‘As seen from this igure, the loop gain tends to approuch that of LOR as the value of q increases. As sven in Fig. 7.43, for q = 10 the “recovered” gain margin is GM = 11.1 = 20.9 db and PM = 55.06". Sample MATLAB statements to carry out the above LTR design procedure are shown below. sy50 = ss(FG.H); Ht =[1 0} syS = SSI J) legspacet-1,3, 1000} 0; ‘ho: Qe hott IKi=laiF6.07) syst = ssfF.G,K.0} [magic ,phasgkw] = bode(syst,w): 88k = ss(aa.bb,co.d0; ‘sysgk = series\sy50,5y5h): [maggk.phsak.l = bodetsysgk,w) [arphm.wea.wep] = margin(magak.phsgk.w) loglog(w{maggki(:) maggki): ssemiogx(ve[phasgk1() phsgk() (©) To determine the effect of sensor noise, v,on the actuator activity. we determine the transfer function from 1 to u as shown in Fig, 7-44. For the selected value of LTR design parameter, q = 10, we have vw =as) i) = 5 vo) "= Te panGey ~ ¥ =155.5602(5 40.6428) FISSS69-4 Dis? + 155.566 7 99. 99E (One reasonable measure of the effect of the sensor noise on the actuator activity i the root-mean-square (RMS) value ofthe contro, ,due tothe additive noise, ». The RMS. value ofthe control may be computed as follows: te toae (Ef merar) rs) Section 7.7 Loop Transfer Recovery (LTR) $73 Figure 7.44 yep Gacapapten oct *—E)—BO}-f a9 }yor L—__@y where 7) is the signal duration. Assuming white gaussian noise v, the RMS value of the control can also be determined analytically (see Boyd and Barratt, 1991). The closed-loop Simulink diagram with band-limited white sensor noise excitation is shown in Fig. 7.45. The values of the RMS control were computed for different values of the [UTR design parameter, using the Simulink simulations and are tabulated in Table 7.2. Simulink simulation Tho results suggest increased vulnerability due to actuator wear as q is increased. TABLE 7.2 Computed RMS Control for Various 300 705216 Figure 7.45 Simutink block olagram for om ‘To workspaces ji Soop? Bang limited white noise Sate-space cooler Art Ba y=Gr+ bu =| ig "To workspace’ 574 Chapter? State-Space Design Figure 7.46 Possible lacations for inoducing the command input: (a) compensation in the feedback path; (b) compensation in the feedforward path 7.8. Introduction of the Reference Input with the Estimator ‘The controller obtained by combining the contro! law studied in Section 7. with the estimator discussed in Section 7.6 is essentially a regulator design, ‘This means that the characteristic equationsof the control and the estimator are chosen for good disturbance rejection—that is, to give satisfactory transients to disturbances such as 20(t). However, this design approach does not consider a reference input, nor does it provide for command following, which is evidenced bya good transient response of the combined system to command changes. In ‘general, good disturbance rejection and good command following both need to be taken into account in designing a control system. Good command following is done by properly introducing the reference input into the system equations. Let us repeat the plant and controller equations for the full-order estimator; the reduced-order case is the same in concept, differing only in detail: Plant: x= Fx +Gu. (7.1824) y= He: (7.182b) Controller : F — GK - Lk +Ly (7.s3a) w= KS, (7.183) Figure 7.46 shows two possibilities for introducing the command input r into the system. This igure ilestrates the general issue of whether the compensation Fa Comprar tat tG) Compensator o Section 7.8 Introduction of the Reference Input with the Estimator 575 should be put in the feedback or feedforward path. The response of the system to command inputs is different, depending on the configuration, because the zeros of the transfer functionsare different. The closed-loop poles are identical, however, as can be easily verified by letting r = 0 and noting that the systems are then identical ‘The difference in the responses of the two configurations can be scen quite easily. Consider the effect of a step input in r. In Fig, 7.46(a) the step will excite the estimator in precisely the same way that it excites the plant; thus the estimator error will remain zero during and after the step. This means that the estimator dynamics are not excited by the command input, so the transfer func tion from r to y must have zerosat the estimator pole locations that cancel those poles. As aesult,a step command will excite system behavior that is consistent With the control poles alone—that is, with the roots of det(sI — F + GK) = 0. In Fig. 7.46(b) a step command in r enters directly only into the estima- tor, thus causing an estimation error that decays with the estimator dynamic characteristics in addition to the response corresponding to the control poles. Therefore, a step command will excite system behavior consistent with both control roots and estimator roots—that is, the roots of det(sI = F + GK) -det(s1 —F + LH) For this reason the configuration shown in Fig, 7.46(a) is typically the su- perior way to command the system where W is found using Eqs. (7.89)-(7.91). In Section 7.8.1, we will show a general structure for introducing the ref- erence input with three choices of parameters that implement either the feed- forward or the feedback case. We will analyze the three choices from the point of view of the system zeros and the implications the zeros have for the sys tem transient response. Finally, in Section 7.8.2 we will show how to select the remaining parameter to eliminate constant errors. 7.8.1 A General Structure for the Reference Input Given reference input r(t), the most general linear way tintroducer into the system equations is to add terms proportional to it in the controller equations We can do this by adding Wr 10 Eg, (7.183b) and Mr to Eq, (7.183a).. Note that inthis case, Nisa scalar and M isan n x 1 vector. With these additions the controller equations become 8 = (F—GK- LAS + Ly + Mr, (7.184a) w= K+ Nr, 7.184) The block diagram is shown in Fig. 747(a). The alternatives shown in Fig, 746 correspond to different choices of Mand N’. Because r(1) is an external signal, itis clear that neither M nor W affects the characteristic equation of the com- bined controller-estimator system. In transfer-function terms, the selection of

Potrebbero piacerti anche